Author Archives: Hung Nguyen

Đề thi và đáp án kì thi chọn đội tuyển thi Quốc gia trường Phổ thông Năng khiếu năm học 2016 -2017

Đề thi

Ngày thi thứ nhất

Bài 1. Tìm tất cả $a$ để dãy số $(u_n)$ hội tụ, biết $u_1=a$ và $\forall n\in \mathbb{N}^*$ thì:
$$u_{n+1}=\left\{\begin{array}{l}
2u_n-1\ \text{nếu $u_n>0$,}\\
-1\ \text{nếu $-1\le u_n\le 0$,}\\
u_n^2+4u_n+2\ \text{nếu $u_n<-1$.}
\end{array} \right.$$

Bài 2. Tìm số nguyên dương $k$ nhỏ nhất để bất đẳng thức $$x^ky^kz^k(x^3+y^3+z^3)\le 3$$
luôn đúng với mọi số thực dương $x,y,z$ thoả mãn điều kiện $x+y+z=3$.

Bài 3. Cho hàm số $f:\mathbb N^* \rightarrow \mathbb N^*$ thoả mãn hai điều kiện sau:

i)  $f$ là hàm tăng thật sự trên $\mathbb N^*$.

ii) $f(2n)=2f(n)\ \forall n\in \mathbb N^*$.

a) Giả sử $f(1)=3$ và $p>3$ là số nguyên tố. Chứng minh rằng tồn tại số nguyên dương $n$ sao cho $f(n)$ chia hết cho $p$.
b) Cho $q$ là số nguyên tố lẻ. Hãy xây dựng một hàm $f$ thoả mãn các điều kiện của bài toán mà $f(n)$ không chia hết cho $q$ với mọi $n$ nguyên dương.

Bài 4. Cho tam giác $ABC$ có góc $\angle BAC$ tù và $AH\perp BC$ ($H$ nằm trên $BC$). Điểm $M$ thay đổi trên cạnh $AB$. Dựng điểm $N$ sao cho $\Delta BMN\sim \Delta HCA$, với $H$ và $N$ nằm khác phía đối với đường thẳng $AB$.

a) Gọi $CM$ cắt đường tròn ngoại tiếp tam giác $BMN$ tại $K$. Chứng minh rằng $NK$ luôn đi qua một điểm cố định.
b) Gọi $NH$ cắt $AC$ tại $P$. Dựng điểm $Q$ sao cho $\triangle HPQ\sim \Delta HNM$, với $Q$ và $M$ nằm khác phía đối với đường thẳng $NP$. Chứng minh rằng $Q$ luôn thuộc một đường thẳng cố định.

Ngày thi thứ hai

Bài 5. Với mỗi số nguyên dương $n$, tồn tại duy nhất số tự nhiên $a$ thoả mãn điều kiện $a^2\le n<(a+1)^2$. Đặt $\Delta_n=n-a^2$.

a) Tìm giá trị nhỏ nhất của $\Delta_n$ khi $n$ thay đổi và luôn thoả mãn $n=15m^2$ với $m$ là số nguyên dương.
b) Cho $p,q$ là các số nguyên dương và $d=5(4p+3)q^2$. Chứng minh rằng $\Delta_d\ge 5$.

Bài 6.  Với các số nguyên $a,b,c,d$ thoả mãn $1\le a<b<c<d$, ký hiệu:
$$T(a,b,c,d)={{x,y,z,t}\subset \mathbb{N}^*\mid 1\le x<y<z<t,\ x\le a,y\le b,z\le c,t\le d}$$

a) Tình số phần tử của $T(1,4,6,7)$.
b) Cho $a=1$ và $b\ge 4$. Gọi $d_1$ là số phần tử của $T(a,b,c,d)$ chứa $1$ và không chứa $2$; $d_2$ là số phần tử chứa $1,2$ và không chứa $3$; $d_3$ là số phần tử chứa $1,2,3$ và không chứa $4$. Chứng minh rằng $d_1\ge 2d_2-d_3$. Đẳng thức xảy ra khi nào ?

Bài 7. Trong một hệ thống máy tính, một máy tính bất kỳ có kết nối trực tiếp với ít nhất $30\%$ máy tính khác của hệ thống. Hệ thống này có một chương trình cảnh báo và ngăn chặn khá tốt, do đó khi một máy tính bị virus, nó chỉ có đủ thời gian lây cho các máy tính được kết nối trực tiếp với nó. Chứng minh rằng dù vậy, kẻ tấn công vẫn có thể chọn hai máy tính của hệ thống mà nếu thả virus vào hai máy đó, ít nhất $50\%$ máy tính của hệ thống sẽ bị nhiễm virus.

Bài 8. Cho tam giác $ABC$ nhọn. Đường tròn $(I)$ có tâm $I$ thuộc cạnh $BC$ và tiếp xúc với các cạnh $AB,AC$ lần lượt tại $E,F$. Lấy $M,N$ bên trong tứ giác $BCEF$ sao cho $EFNM$ nội tiếp $(I)$ và các đường thẳng $MN,EF,BC$ đồng quy. Gọi $MF$ cắt $NE$ tại $P$, $AP$ cắt $BC$ tại $D$.

a) Chứng minh rằng $A,D,E,F$ cùng thuộc một đường tròn.
b) Lấy trên các đường thẳng $BN,CM$ các điểm $H,K$ sao cho $\angle ACH=\angle ABK=90^\circ$. Gọi $T$ là trung điểm $HK$. Chứng minh rằng $TB=TC$.

Hết

Lời giải

Lời giải

Bài 1. 

  • Nếu $a>1$, bằng quy nạp đơn giản, ta có $u_n>1\ \forall n\in \mathbb N^*$ và
    $$u_n = 2^{n-1}(a-1)+1, \ \forall n\in \mathbb N^*.$$
    Do $a>1$, cho $n\rightarrow +\infty$ thì $u_n\rightarrow +\infty$. Từ đó $(u_n)$ không hội tụ.
  •  Nếu $a=1$ thì $u_n=1\ \forall n\in \mathbb N^*$ hay $(u_n)$ hội tụ về $1$.
  • Nếu $0<a<1$, ta sẽ chứng minh rằng $(u_n)$ có ít nhất một số hạng không dương. Thật vậy, giả sử $u_n>0\ \forall n\in \mathbb N^*$ thì theo trường hợp đầu tiên, ta có:
    $$u_n = 2^{n-1}(a-1)+1\ \forall n\in \mathbb N^*$$
    Do $a>1$, cho $n\rightarrow +\infty$ thì $u_n\rightarrow -\infty$, trái với việc $u_n>0\ \forall n, \in \mathbb N^*$. Từ đó điều giả sử là sai hay phải tồn tại $k\in \mathbb N^*$ sao cho $u_k>0$ và $u_{k+1}\le 0$. Với cách chọn chỉ số $k$ như vậy, ta có:
    $$-1\le 2u_k-1=u_{k+1}\le 0$$
    Khi đó $u_{k+2}=0$. Bằng quy nạp thì $u_n=-1\ \forall n\in \mathbb N^*, n\ge k+2$. Điều này dẫn đến $(u_n)$ hội tụ về $-1$.
  • Nếu $-1\le a\le 0$, từ giả thiết thì $u_2=-1$. Bằng quy nạp thì $u_n=-1\ \forall n\in \mathbb N^*, n\ge 2$ hay $(u_n)$ hội tụ về $-1$.
  • Nếu $-2<a<-1$, ta có:
    $$u_2-u_1=a^2+3a+2=(a+2)(a+1)<0$$
    Khi đó thì $u_2<u_1<-1$. Lại có $u_2=(a+2)^2-2\ge -2$ nên $-2<u_2<-1$. Bằng quy nạp, ta có $(u_n)$ là dãy giảm và $-2<u_n<-1$ nên $(u_n)$ hội tụ.
  • Nếu $-2-\sqrt{3}\le a\le -2$ thì $u_2=a^2-4a+2$ và dễ có được:
    $$-1\le a^2-4a+2\le 1$$
    Theo các trường hợp đã xét, dãy số $(u_n)$ hội tụ.
  • Nếu $a<-2-\sqrt{3}$, bằng vài tính toán, ta có $u^2=a^2-4a+2>1$.\\
    Theo trường hợp đầu tiên, dãy số $(u_n)$ không hội tụ.Vậy dãy số $(u_n)$ hội tụ khi và chỉ khi $-2-\sqrt{3}\le a\le 1$.]

Bài 2. Ta sẽ chứng minh rằng $k=3$ là số nguyên dương nhỏ nhất thoả mãn bài toán. Trước hết, chọn $x=y=\dfrac{3}{4},z=\dfrac{3}{2}$ thì ta phải có:
$$\left(\frac{3}{4}\right)^{2k}\cdot \left(\frac{3}{2}\right)^{k}\left(2\cdot\left(\frac{3}{4}\right)^{3}+\left(\frac{3}{2}\right)^3\right)\le 3$$
Dễ thấy đánh giá trên chỉ đúng nếu $k\ge 3$. Ta đưa về chứng minh rằng:
$$x^3y^3z^3(x^3+y^3+z^3)\le 3.$$
Không mất tính tổng quát, giả sử $x\ge y\ge z$ thì $z \le 1$. Ta có:
$$\begin{aligned} & x^3+y^3=(x+y)^3-3xy(x+y)=(3-z)^3-3xy(x+y) \text{ hay} \\
&(3-z)^3 + z^3 \le \frac{3}{x^3y^3z^3}+3xy(x+y). \end{aligned} $$
Khai triển và thu gọn, bất đẳng thức trở thành:
$$3z^2-9z+9 \le \frac{1}{x^3y^3z^3}+x^2y+xy^2.$$
Theo bất đẳng thức AM-GM, ta có vế phải của bất đẳng thức trên sẽ không nhỏ hơn $\frac{3}{z}$. Từ đây ta chỉ cần chứng minh rằng $$3z^2-9z+9 \le \dfrac{3}{z} \text{ hay } 3(z-1)^3 \le 0, \text{ đúng.}$$
Vậy $k=3$ là hằng số nguyên dương nhỏ nhất thoả mãn bài toán.

Bài 3.

(a) Đặt $A = \\{f(n+1)-f(n)|n \in\mathbb{N^{*}}\\}$.

Vì $f$ là hàm số tăng thực sự trên $\mathbb{N^{*}}$ nên $A\subset \mathbb{N^{*}}$.

Khi đó phải tồn tại $k=\min A$ và tồn tại $n\in \mathbb N^*$ để $k=f(n+1)-f(n)$.

Khi đó:
$$f(2n+2) – f(2n) = 2f(n+1) – 2f(n) = 2k.$$

Lại có $f(2n+2) – f(2n+1),f(2n+1) – f(2n)\ge k$ nên

$$f(2n+2)-f(2n+1)+f(2n+1)-f(2n)\ge 2k.$$
Từ đây ta phải có $f(2n+2) – f(2n+1)=f(2n+1) – f(2n)= k$. Bằng quy nạp theo $m$, ta chứng minh được

$$f(2^mn+t) = 2^mf(n)+tk\ \forall t,m\in \mathbb N, t\le m.$$

Lại có $f(1)=3,f(2)=6$ nên $k \le 3<p$ hay $(k, p)=1$. \medskip

Xét $p$ số nguyên dương sau:
$$f(2^pn), f(2^pn+1), f(2^pn+2),\ldots, f(2^pn+p-1)$$
lập thành một cấp số cộng có công sai $k$ nên là một hệ thặng dư đầy đủ modulo $p$. Từ đó phải tồn tại một số hạng chia hết cho $p$. \medskip

(b) Ta xây dựng một hàm số $f$ với các điều kiện như sau:

$f(1) = 2^a > q (a\in\mathbb{N^{*}},$

$f(2n)=2f(n)\ \forall n\in \mathbb{N^{*}},$

$f(2n+1)=f(2n)+q\ \forall n\in \mathbb{N^{*}}.$

Ta chứng minh rằng hàm số $f$ vừa xây dựng thỏa mãn bài toán. \medskip

Trước hết ta chứng minh rằng $f$ là hàm tăng thực sự, cụ thể là:
$$f(n+1) – f(n) \geq q\ \forall n\in \mathbb{N^{*}}.$$
Với $n = 1$, ta có $f(2)-f(1) = 2.2^a – 2^a = 2^a > q$. Giả sử khẳng định cần chứng minh đúng đến $n=k$. Xét các khả năng sau:

Nếu $k$ là số chẵn, ta có $f(k+1)=f(k)+q$ thỏa mãn yêu cầu.
Nếu $k$ là số lẻ, ta có:
$$f(k+1)= 2f\left(\dfrac{k+1}{2}\right) \geq 2\left(f\left(\dfrac{k-1}{2}\right)+q\right)= f(k-1)+2q.$$
Lại có $f(k)=f(k-1)+q$ nên $f(k+1)\ge f(k)+q$.

Theo nguyên lý quy nạp, ta có $f(n+1) – f(n) \geq q\ \forall n\in \mathbb{N^{*}}$. \medskip

Bây giờ ta chứng minh rằng không tồn tại $n$ để $q \mid f(n)$. Trước hết thì $f(1) = 2^a$ không chia hết cho $q$. Giả sử điều này đúng đến $n=k$. Xét các khả năng sau:

Nếu $k$ chẵn thì $f(k+1)=f(k)+q$ không chia hết cho $q$.
Nếu $k$ lẻ thì $f(k+1)= 2f\left(\dfrac{k+1}{2}\right)$ không chia hết cho $q$.

Theo nguyên lý quy nạp, $f(n)$ không chia hết cho $q$ với mọi $n\in \mathbb{N^{*}}$.
Các điều kiện đã được kiểm tra đầy đủ.

BÀI 4.

(a) Ta sẽ chứng minh rằng $AD\perp BC$. Gọi $X$ là điểm đồng quy của $EF,MN,BC$. Do $AE,AF$ tiếp xúc với $(I)$ nên $EF$ là đường đối cực của $A$ đối với $(I)$. Ta có $X\in EF$ nên theo định lý La Hire, điểm $A$ sẽ nằm trên đường đối cực của $X$ đối với đường tròn $(I)$. \medskip

Lại có $P$ là giao điểm của $EN,FM$ nên $P$ nằm trên đường đối cực của $X$ đối với $(I)$. Vì thế nên $AP$ là đường đối cực của $X$ đối với $(I)$ hay $AP\perp BC$. Do đó $$\angle ADI=\angle AEI=\angle AFI=90^\circ.$$
Vậy $A,D,E,F$ cùng thuộc một đường tròn.

(b) Gọi $S$ là giao điểm của $BN,CM$. Xét hai tam giác $PEF,SBC$ có $PE$ cắt $SB$ tại $N$, $PF$ cắt $SC$ tại $M$, $EF$ cắt $BC$ tại $X$ và $X,M,N$ thẳng hàng. Theo định lý Desargues thì $PS,EB,FC$ đồng quy. Mặt khác $EB$ cắt $FC$ tại $A$ nên $A,P,S$ thẳng hàng, dẫn đến $S\in AD$. \medskip

Tiếp theo ta sẽ chứng minh rằng $\angle BAK=\angle CAH$. Áp dụng định lý Ceva dạng lượng giác cho tam giác $ABC$ với:

Các đường thẳng $AD,BH,CK$ đồng quy:
$$\frac{\sin\angle DAB}{\sin \angle DAC}\cdot \frac{\sin\angle HBC}{\sin \angle HBA}\cdot \frac{\sin\angle KCA}{\sin \angle KCB}=1$$
Các đường thẳng $AH,BH,CH$ đồng quy:
$$\frac{\sin\angle HAB}{\sin \angle HAC}\cdot \frac{\sin\angle HBC}{\sin \angle HBA}\cdot \frac{\sin\angle HCA}{\sin \angle HCB}=1$$
Các đường thẳng $AK,BK,CK$ đồng quy:
$$\frac{\sin\angle KAB}{\sin \angle KAC}\cdot \frac{\sin\angle KBC}{\sin \angle KBA}\cdot \frac{\sin\angle KCA}{\sin \angle KCB}=1$$

Chú ý rằng do các góc vuông và góc bù nhau nên ta có
$$\frac{\sin\angle HAC}{\sin \angle HAB}=\frac{\sin\angle KAB}{\sin \angle KAC}$$
Từ đó sử dụng công thức cộng cho mẫu thức và biến đổi thì:
$$\tan\angle HAC=\tan\angle KAB$$
Dẫn đến $\angle HAC=\angle KAB$. Cuối cùng, ta sẽ chứng minh $TB=TC$.
Gọi $U,V$ lần lượt là trung điểm của các đoạn $AK,AH$. Ta có:
$$UB=\dfrac{AK}{2}=VT,UT=\dfrac{AH}{2}=VC.$$
Đồng thời, ta cũng có:
$$\angle BUT=\angle BUA-\angle AUT=\angle AVC-\angle AVT=\angle TVC$$
Do đó $\Delta BUT=\Delta TVC$ (c.g.c), vậy nên $TB=TC$.

Bài 5. 

(a) Ta cần tìm $\Delta_n$ nhỏ nhất để phương trình $15m^2 – a^2 = \Delta_n$ có nghiệm nguyên dương. Nhận thấy $15 – 3^2 = 6$ nên $\min \Delta_n\le 6$. Ta chứng minh rằng phương trình trên không có nghiệm nguyên dương với $\Delta_n < 6$. \medskip

Ta có $3\mid a^2 + \Delta_n$. Suy ra $3\mid \Delta_n $ hoặc $3\mid \Delta_n+1$. Mặt khác $5\mid a^2 + \Delta_n$ nên $\Delta_n$ chia $5$ chỉ có thể dư $0,1$ hoặc $4$. \medskip

Từ đó nếu tồn tại $n$ để $\Delta_n< 6$ thỏa mãn bài toán thì $\Delta_n = 5$. Giả sử rằng tồn tại $n$ như thế, ta có $15m^2-a^2=5$ hay $5\mid a$. Đặt $a=5s$ $(s\in \mathbb N^*)$, ta có:
$$3m^2 – 5s^2 = 1.$$
Từ đó thì $$3(m^2+s^2)\equiv 1 \pmod{8} \text{ hay } m^2+s^2\equiv 3 \pmod{8}.$$
Điều này vô lý do $m^2$ chia $8$ dư $0,1,4$. Vậy $\Delta_n$ nhỏ nhất là $6.$ \medskip

(b) Ta có $$5(4p+3)q^2-a^2=\Delta_d.$$ Do $a^2$ chia $5$ dư $0,1,4$ nên $\Delta_d$ chia $5$ dư $0,1,4$. Giả sử rằng có bộ số để $\Delta_d<5$. Xét các khả năng sau:

Nếu $\Delta_d=0$ thì $5(4p+3)q^2=a^2$. Xét bộ số $(q,a)$ với $q+a$ nhỏ nhất. Từ phương trình trên, ta có $a^2+q^2\equiv 0$ (mod $4$) hay $a\equiv q\equiv 0$ (mod $2$).\medskip

Đặt $a=2a_1$ và $q=2q_1$ với $a_1,q_1\in \mathbb N^*$ thì bộ số $(q_1,a_1)$ cũng thoả mãn điều kiện $5(4p+3)q_1^2=a_1^2$. Hơn nữa $q_1+a_1<q+a$, mâu thuẫn.
Nếu $\Delta_d = 1$, ta có $a^2 + 1 = 5(4p+3)q^2$. Do $5(4p+3)\equiv 3$ (mod $4$) nên số này tồn tại một ước nguyên tố $r\equiv 3$ (mod $4$).\\
Do đó $a^2+1\equiv 0$ (mod $r$) hay $r\mid 1$, vô lý.
Nếu $\Delta_d = 4$, chứng minh tương tự, ta cũng có điều mâu thuẫn.

Vậy ta phải có $\Delta_d \ge 5$.

Bài 6.

(a) Với $T(1,4,6,7)$, ta có $x \leq 1$ nên $x =1$. Khi đó ta có $2\le y \le 4$ hay $y\in \{2,3,4\}$. Xét các khả năng sau:

Nếu $y = 2$ thì $3\leq z \leq 6$. Với mỗi giá trị của $z$, ta có thể thu được $7-z$ giá trị của $t$ nên ta có 10 bộ số.
Nếu $y=3$, tương tự ta có $6$ bộ số.
Nếu $y= 4$, tương tự ta có $3$ bộ số.

Vậy có tất cả $19$ bộ số trong $T(1,4,6,7)$. \medskip

(b) Đặt các tập hợp sau:
$$\begin{cases}
T_1 = \{(1, y, z, t)\mid 3\le y \le b, y<z\le c, z<t \leq d \}\\\\
T_2 = \{(1, 2, z, t)\mid 4\le z \le c, z<t\le d \}\\\\
T_3 = \{(1, 2, 3, t)\mid 5\le t \le d \}
\end{cases}.$$
Ta có $d_3 = |T_3| = d – 4$ và
$$d_2=\sum_{z=4}^{c}(d-z)=(c-3)d+\frac{(c+4)(c-3)}{2}.$$
Tiếp theo ta tính $d_1=|T_1|$. Vì $b \ge 4$ nên $y \ge 3$. Xét các khả năng sau

Nếu $y=3$ thì $T(1,3,z,t)=d_2$.
Nếu $y=4$ thì $T(1,4,z,t)=\sum_{z=5}^{c} (d-z)=(c-4)d-\dfrac{(c+5)(c-4)}{2}$. \medskip

Từ đó $d_1\ge d_2+(c-4)d-\dfrac{(c+5)(c-4)}{2}$. Do đó, kết hợp với việc tính được giá trị của $d_2$, khi cộng theo vế thì $d_1+d_3 – 2d_2 \ge 0.$

Vậy $d_1\ge 2d_2-d_3$. Đẳng thức xảy ra khi và chỉ khi $b=4$.

Ngoài lời giải khá “đại số” phía trên, có một lời giải khác cho ý sau của bài toán sử dụng song ánh:

Điểm mấu chốt là phân rã $T_1,T_2,T_3$ thành các nhóm thích hợp và thiết lập được đơn ánh giữa chúng. Với các tập $T_1,T_2,T_3$ định nghĩa như trên, ta viết $T_1$ thành $A\cup B\cup C$ có giao đôi một khác rỗng, trong đó
$$\begin{cases}
A = \{(1, 3, 4, t)\mid 5\le t \le d \}\\\\
B = \{(1, 3, z, t)\mid 5\le z \le c, z<t\le d \}\\\\
C = \{(1, y, z, t)\mid 4\le y\le b, y<z\le c, z<t \le d \}
\end{cases}.$$
Dễ kiểm chứng rằng có song ánh từ $A$ vào $T_3$ nên $|A|=|T_3|=d_3$.
Xét $D=\{(1, 4, z, t)\mid 5\le z \le c, z<t\le d \}$. Dễ kiểm chứng rằng $D\subset C$ và có song ánh từ $D$ vào $B$ nên $|D|=|B|$.
Ta có $A\cup B=\{(1, 3, z, t)\mid 4\le z \le c, z<t\le d \}$. Dễ kiểm chứng rằng có song ánh từ $A\cup B$ vào $T_2$ nên $|A\cup B|=|T_2|=d_2$. Chú ý rằng $A\cap B=\varnothing$ nên $|A|+|B|=d_2$ hay $|B|=d_2-d_3$. Từ đó ta có:
$$d_1=|A|+|B|+|C|\ge |A|+|B|+|D|=d_3+2|B|$$
Vậy $d_1\ge d_3+2(d_2-d_3)=2d_2-d_3$. Đẳng thức xảy ra khi và chỉ khi $b=4$.

Bài 7.

Trước hết ta chứng minh bổ đề sau: Xét một tập con $S$ bất kỳ của tập các máy tính $X$, khi đó tồn tại $1$ máy tính của hệ thống kết nối trực tiếp với ít nhất $30\%$ máy tính của $S$. \medskip

Thật vậy, xét các cặp $(s, x)$ với $s\in S,x\in X$ và $(s,x)$ kết nối trực tiếp với nhau. Khi đó, nếu tính theo $s$ thì số cặp như vậy sẽ không ít hơn $\dfrac{3}{10}|S||X|$. Do đó nếu tính theo $x$ thì sẽ phải tồn tại máy tính $x$ kết nối trực tiếp với ít nhất $\dfrac{3}{10}|S|$. \medskip

Quay trở lại bài toán, \medskip

Giả sử hệ thống có $n$ máy tính. Xét máy tính $A$ bất kỳ. Gọi $S$ là tập hợp các máy tính không kết nối trực tiếp với $A$. Nếu $S=\varnothing$ thì kết quả bài toán là hiển nhiên. Nếu $S\ne \varnothing$ thì theo bổ đề, tồn tại máy tính $B$ kết nối trực tiếp với ít nhất $30\%$ máy tính trong $S$. Ta chứng minh hai máy tính $A$ và $B$ thỏa mãn yêu cầu bài toán. \medskip

Thật vậy, giả sử $A$ kết nối trực tiếp với $k$ máy tính khác. Khi đó, theo cách chọn, $A$ và $B$ sẽ kết nối trực tiếp với ít nhất
$$k + 0,3(n-k) = 0,7k + 0,3n \ge 0,7\cdot 0,3n + 0,3n = 0,51n.$$
Từ đây ta có được kết luận của bài toán.

Bài 8.

(a) Ta sẽ chứng minh rằng $AD\perp BC$. Gọi $X$ là điểm đồng quy của $EF,MN,BC$. Do $AE,AF$ tiếp xúc với $(I)$ nên $EF$ là đường đối cực của $A$ đối với $(I)$. Ta có $X\in EF$ nên theo định lý La Hire, điểm $A$ sẽ nằm trên đường đối cực của $X$ đối với đường tròn $(I)$. \medskip

Lại có $P$ là giao điểm của $EN,FM$ nên $P$ nằm trên đường đối cực của $X$ đối với $(I)$. Vì thế nên $AP$ là đường đối cực của $X$ đối với $(I)$ hay $AP\perp BC$. Do đó $$\angle ADI=\angle AEI=\angle AFI=90^\circ.$$
Vậy $A,D,E,F$ cùng thuộc một đường tròn.

(b) Gọi $S$ là giao điểm của $BN,CM$. Xét hai tam giác $PEF,SBC$ có $PE$ cắt $SB$ tại $N$, $PF$ cắt $SC$ tại $M$, $EF$ cắt $BC$ tại $X$ và $X,M,N$ thẳng hàng. Theo định lý Desargues thì $PS,EB,FC$ đồng quy. Mặt khác $EB$ cắt $FC$ tại $A$ nên $A,P,S$ thẳng hàng, dẫn đến $S\in AD$. \medskip

Tiếp theo ta sẽ chứng minh rằng $\angle BAK=\angle CAH$. Áp dụng định lý Ceva dạng lượng giác cho tam giác $ABC$ với:

Các đường thẳng $AD,BH,CK$ đồng quy:
$$\frac{\sin\angle DAB}{\sin \angle DAC}\cdot \frac{\sin\angle HBC}{\sin \angle HBA}\cdot \frac{\sin\angle KCA}{\sin \angle KCB}=1$$
Các đường thẳng $AH,BH,CH$ đồng quy:
$$\frac{\sin\angle HAB}{\sin \angle HAC}\cdot \frac{\sin\angle HBC}{\sin \angle HBA}\cdot \frac{\sin\angle HCA}{\sin \angle HCB}=1$$
Các đường thẳng $AK,BK,CK$ đồng quy:
$$\frac{\sin\angle KAB}{\sin \angle KAC}\cdot \frac{\sin\angle KBC}{\sin \angle KBA}\cdot \frac{\sin\angle KCA}{\sin \angle KCB}=1$$

Chú ý rằng do các góc vuông và góc bù nhau nên ta có
$$\frac{\sin\angle HAC}{\sin \angle HAB}=\frac{\sin\angle KAB}{\sin \angle KAC}$$
Từ đó sử dụng công thức cộng cho mẫu thức và biến đổi thì:
$$\tan\angle HAC=\tan\angle KAB$$
Dẫn đến $\angle HAC=\angle KAB$. Cuối cùng, ta sẽ chứng minh $TB=TC$.

Gọi $U,V$ lần lượt là trung điểm của các đoạn $AK,AH$. Ta có:
$$UB=\dfrac{AK}{2}=VT,UT=\dfrac{AH}{2}=VC.$$
Đồng thời, ta cũng có:
$$\angle BUT=\angle BUA-\angle AUT=\angle AVC-\angle AVT=\angle TVC$$
Do đó $\Delta BUT=\Delta TVC$ (c.g.c), vậy nên $TB=TC$.

Đề thi và đáp án kì thi chọn đội tuyển thi Quốc gia trường Phổ thông Năng khiếu năm học 2015 -2016

Đề bài

Ngày thi thứ nhất

Bài 1.  Cho tập hợp:
$$A=\left\{ n\in \mathbb{N}\mid 1\le n\le 2015,\gcd (n,2016)=1 \right\}.$$
Hỏi có bao nhiêu số nguyên $a\in A$ sao cho tồn tại số nguyên $b$ mà $a+2016b$ là số chính phương?
Bài 2. Cho $a,b,c,d$ là các số thực thỏa mãn điều kiện:
$$\left\{\begin{array}{l}
a^2\le 1\\
a^2+b^2\le 5\\
a^2+b^2+c^2\le 14\\
a^2+b^2+c^2+d^2\le 30
\end{array} \right..$$

a)Chứng minh rằng $a+b+c+d\le 10$.
b) Chứng minh rằng $ad+bc\le 10$.

Bài 3.  Tìm tất cả các hàm số $f:\mathbb{R}\to \mathbb{R}$ thỏa mãn:
$$f\left( x-2f(y) \right)=5f(x)-4x-2f(y), \, \ \forall x,y\in \mathbb R.$$
Bài 4. Cho đường tròn $k$ và các điểm $B,C$ thuộc đường tròn sao cho $BC$ không phải là đường kính; $I$ là trung điểm $BC$. Điểm $A$ di động trên cung lớn $BC$ của $k$. Gọi $(\mathcal I_1)$ là đường tròn qua $I$ và tiếp xúc với $AB$ tại $B$, $(\mathcal I_2)$ là đường tròn qua $I$ và tiếp xúc với $AC$ tại $C$. Các đường tròn $(\mathcal I_1), (\mathcal I_2)$ cắt nhau tại $D$.

a) Chứng minh rằng đường tròn ngoại tiếp tam giác $AID$ luôn đi qua một điểm cố định.
b) Gọi $K$ là trung điểm $AD$, $E$ là tâm đường tròn qua $K$ và tiếp xúc với $AB$ tại $A$, $F$ là tâm đường tròn qua $K$ và tiếp xúc với $AC$ tại $A$. Chứng minh rằng góc $\angle EAF$ có số đo không đổi.

Ngày thi thứ hai

Bài 5. Cho dãy số $(x_n)$ được xác định bởi $x_n=\dfrac{1}{n\cos \frac{1}{n}}\ \forall n\in \mathbb N^*$. Tính giới hạn:
$$\lim_{n \to +\infty} \frac{{{x}_{1}}+{{x}_{3}}+{{x}_{5}}+\cdots+{{x}_{2n-1}}}{{{x}_{2}}+{{x}_{4}}+{{x}_{6}}+\cdots +{{x}_{2n}}}.$$
Bài 6. Tìm các giá trị của $b$ sao cho tồn tại $a$ để hệ phương trình sau có nghiệm $(x,y)$:
$$\left\{\begin{array}{l}
(x-1)^2+(y+1)^2=b \\
y=x^2+(2a+1)x+a^2
\end{array} \right..$$
Bài 7. Cho $n$ là số nguyên dương, $n\ge 2$ và $X=\left\{ 1,2,3,\ldots,n \right\}$. Gọi ${{A}_{1}},{{A}_{2}},\ldots,{{A}_{m}}$ và ${{B}_{1}},{{B}_{2}},\ldots,{{B}_{m}}$ là hai dãy các tập con khác rỗng của $X$ thỏa mãn điều kiện sau: với mỗi $i,j\in \left\{ 1,2,3,\ldots,m \right\}$, ${{A}_{i}}\cap {{B}_{j}}=\varnothing $ nếu và chỉ nếu $i=j.$
a) Chứng minh rằng với mỗi hoán vị $(x_1,x_2,\ldots,x_n)$ của tập hợp $X$, có không quá một cặp tập hợp $(A_i,B_i)$ với $i=1,2,3,\ldots,m$ sao cho nếu $x_k\in A_i$ và $x_l\in B_i$ thì ta phải có $k<l$.
b) Gọi $a_i,b_i$ lần lượt là số phần tử của tập hợp $A_i,B_i$ với $i=1,2,3,\ldots,m$. Chứng minh rằng:
$$\sum_{i=1}^{m}{\dfrac{1}{C_{{a_i+b_i}}^{a_i}}}\le 1.$$

Bài 8.  Cho tam giác $ABC$ nhọn nội tiếp đường tròn tâm $O$. Đường tròn tâm $I$ đi qua $B,C$ lần lượt cắt các tia $BA,CA$ tại $E,F.$

a) Giả sử các tia $BF,CE$ cắt nhau tại $D$ và $T$ là tâm đường tròn $(AEF)$. Chứng minh rằng $OT\parallel ID.$
b) Trên $BF,CE$ lần lượt lấy các điểm $G,H$ sao cho $AG\perp CE,AH\perp BF.$ Các đường tròn $(ABF),(ACE)$ cắt $BC$ tại $M,N$ khác $B,C$ và cắt $EF$ tại $P,Q$ khác $E,F$. Gọi $K$ là giao điểm của $MP,NQ$. Chứng minh rằng $DK\perp GH$.

Hết

Giải

Bài 1.

Cho số nguyên dương $n>1$, ta quy ước gọi một số nguyên dương $a < n$ là thặng dư chính phương theo modulo $n$ nếu $\gcd(a,n)=1$ và tồn tại số nguyên $x$ sao cho $a\equiv {{x}^{2}} \pmod n .$ \medskip

Đặt $s(n)$ là số các số như thế. Ta sẽ chứng minh hai bổ đề dưới đây: \medskip

Bổ đề 1. Cho $p$ là số nguyên tố và $k$ là số nguyên dương. Khi đó:

Nếu $p=2$ thì $s({{2}^{k}})={{2}^{\max (k-3,0)}}$.
Nếu $p>2$ thì $s({{p}^{k}})=\dfrac{{{p}^{k}}-{{p}^{k-1}}}{2}$.

Bổ đề 2. $s(n)$ là hàm nhân tính, tức là $s(ab)=s(a)s(b)$ với $\gcd(a,b)=1$. \medskip

Thật vậy, \medskip

Trước hết, ta biết rằng $s(p)=\frac{p-1}{2}$ với $p$ là số nguyên tố lẻ. Ta sẽ tính $s({{p}^{k}})$ với $k\in {{\mathbb{Z}}^{+}}$. Xét một thặng dư chính phương $a$ của $p$, khi đó tồn tại $x$ sao cho $$a\equiv {{x}^{2}}\pmod{p}.$$ Đặt $a={{x}^{2}}+pq$ thì hiển nhiên $$a\equiv {{x}^{2}}+pq \pmod {{p}^{k}}\Leftrightarrow a-pq\equiv {{x}^{2}} \pmod {{p}^{k}}$$ và khi đó, ta có ${{p}^{k-1}}$ cách chọn $q$ để các số $a-pq$ là các thặng dư chính phương theo modulo ${{p}^{k}}$. Suy ra
$$s({{p}^{k}})={{p}^{k-1}}s(p)=\frac{{{p}^{k}}-{{p}^{k-1}}}{2}.$$ Xét số nguyên tố $p=2$, với $k=1,2,3,$ dễ dàng kiểm tra được $s({{2}^{k}})=1$. \medskip

Ta xét $k\ge 4$, tương tự trên, ở bước chọn $q$, ta chỉ có 2 cách nên $s({{2}^{k}})=2s({{2}^{k-1}})$. Từ đó bằng quy nạp, ta có được $$s({{2}^{k}})={{2}^{k-3}},k\ge 4.$$ Tiếp theo, xét hai số $a,b$ nguyên dương nguyên tố cùng nhau. Gọi $A$ là tập hợp các thặng dư chính phương theo modulo $ab$ và $B$ là tập hợp các số là thặng dư chính phương chung của $a,b.$ \medskip

Nếu $x\in A$ thì tồn tại $y$ sao cho $x\equiv {{y}^{2}} \pmod{ab}$. Rõ ràng khi đó,
$$x\equiv {{y}^{2}}\pmod a, \, x\equiv {{y}^{2}}\pmod b$$
(chú ý rằng nếu $x>a$, ta có thể chọn ${x}'$ sao cho ${x}'<a$ và $x\equiv {x}'\pmod a$; tương tự với $b$).
Do đó, $x\in B$, tức là $x\in A\Rightarrow x\in B$ nên $\left| A \right|\le \left| B \right|$. \medskip

Tiếp theo, xét $x\in B$. Khi đó tồn tại $r,s$ sao cho
$x\equiv {{r}^{2}}\pmod a,\text{ }x\equiv {{s}^{2}}\pmod b$.
Theo định lý thặng dư Trung Hoa, tồn tại số nguyên $z$ sao cho $$z\equiv r\pmod a, \, z\equiv s\pmod b.$$ Khi đó $$x\equiv {{z}^{2}}\pmod a, \, x\equiv {{z}^{2}}\pmod b$$ nên
$$x\equiv {{z}^{2}} \pmod{ab}.$$ Do đó: $x\in A$, tức là $x\in B\Rightarrow x\in A$ nên $\left| A \right|\ge \left| B \right|$. Từ đây ta có $$\left| A \right|=\left| B \right| \text{ hay } s(a)s(b)=s(ab).$$ Vậy $s(n)$ là hàm nhân tính. \medskip

Các bổ đề đều được chứng minh. \medskip

Trở lại bài toán, ta thấy rằng $2016={{2}^{5}}\cdot {{3}^{2}}\cdot 7.$ Rõ ràng bài toán yêu cầu đếm số thặng dư chính phương theo modulo $2016$.
Theo bổ đề 2 thì $$s(2016)=s({{2}^{5}})s({{3}^{2}})s(7).$$ Theo bổ đề 1 thì $$s({{2}^{5}})={{2}^{2}}=4,s({{3}^{2}})=\frac{{{3}^{2}}-3}{2}=3,s(7)=\frac{7-1}{2}=3.$$ Do đó, số các số $a$ cần tìm là $4\cdot 3\cdot 3=36.$

Bài 2. 

(a) Dự đoán dấu bằng xảy ra khi $a=1,b=2,c=3,d=4$ nên ta có các đánh giá sau $
{{a}^{2}}+1\ge 2a
{{b}^{2}}+4\ge 4b
{{c}^{2}}+9\ge 6c
{{d}^{2}}+16\ge 8d
$

Do đó, ta có
$24(a+b+c+d)\le 3({{d}^{2}}+16)+4({{c}^{2}}+9)+6({{b}^{2}}+4)+12({{a}^{2}}+1)$
$=3{{d}^{2}}+4{{c}^{2}}+6{{b}^{2}}+12{{a}^{2}}+120 $

$=3({{a}^{2}}+{{b}^{2}}+{{c}^{2}}+{{d}^{2}})+({{a}^{2}}+{{b}^{2}}+{{c}^{2}})+2({{a}^{2}}+{{b}^{2}})+6{{a}^{2}}+120$

$\le 3\cdot 30+14+2\cdot 5+6\cdot 1+120=240$
Suy ra $a+b+c+d\le 10.$ \medskip

(b) Ta có $$16{{a}^{2}}+{{d}^{2}}\ge 8ad \text{ và } 9{{b}^{2}}+4{{c}^{2}}\ge 12bc.$$

Từ đó suy ra

$24(ad+bc)\le 3(16{{a}^{2}}+{{d}^{2}})+2(9{{b}^{2}}+4{{c}^{2}})$
$=3({{a}^{2}}+{{b}^{2}}+{{c}^{2}}+{{d}^{2}})+5({{a}^{2}}+{{b}^{2}}+{{c}^{2}})+10({{a}^{2}}+{{b}^{2}})+30{{a}^{2}}$
$\le 3\cdot 30+5\cdot 14+10\cdot 5+30\cdot 1=240$
Suy ra $ad+bc\le 10.$

Bài 3.

Gọi () là điều kiện đề bài cho.
Trong (
) thay $x=y=0$, ta có $$f(-2f(0))=3f(0).$$ Đặt $f(0)=a$ thì $f(-2a)=3a$.
Trong () thay $x=0$ và $y=-2a$, ta có $$f(-2f(-2a))=5a-2f(-2a)\Leftrightarrow f(-6a)=-a.$$ Trong (), thay $x=-2a,y=-6a$, ta có
$$\begin{aligned}
& f(-2a-2f(-6a))=5f(-2a)-4x-2f(-6a) \\
& \Leftrightarrow f(0)=15a+8a+2a.
\end{aligned}$$ Từ đây ta có $a=25a$ nên $a=0,$ tức là $f(0)=0$. \medskip

Trong $(*),$ thay $y=0$, ta có $$f(x)=5f(x)-4x\Leftrightarrow f(x)=x.$$ Thử lại ta thấy thỏa. Vậy hàm số cần tìm chính là $f(x)=x,\forall x\in \mathbb{R}.$

Bài 4.

(a) Gọi $O$ là tâm của đường tròn $k.$ Không mất tính tổng quát, giả sử tia $AD$ nằm giữa hai tia $AO,AB,$ các trường hợp còn lại tương tự.

Ta có: $$\angle IDB=\angle ABC,\angle IDC=\angle ACB$$ nên $$\angle BAC+\angle BDC=\angle BAC+\angle ABC+\angle ACB=180{}^\circ .$$ Do đó, tứ giác $ABDC$ nội tiếp hay $D\in (O).$ Ta thấy $$\begin{aligned}
& \angle DAO+\angle OID \\
& =\angle BAC-(\angle DAB+\angle OAC)+360{}^\circ -(90{}^\circ +\angle DIC) \\
& =\angle BAC-\left( \angle ICD+90{}^\circ -\angle ABC \right)+270{}^\circ -\angle DIC \\
& =\angle BAC+\angle ABC-(\angle ICD+\angle DIC)+180{}^\circ \\
& =(180{}^\circ -\angle ACB)-\left( 180{}^\circ -\angle IDC \right)+180{}^\circ \\
& =\angle IDC-\angle ACB+180{}^\circ =180{}^\circ.
\end{aligned} $$

Do đó, $AOID$ nội tiếp hay đường tròn $(AID)$ đi qua $O$ cố định. \medskip

(b) Ta có: $$\angle EAC=90{}^\circ -\angle BAC,\angle FAB=90{}^\circ -\angle BAC$$ nên
$$\angle EAF=180{}^\circ -2\angle BAC+\angle BAC=180{}^\circ -\angle BAC.$$

Do đó, góc $\angle EAF$ có số đo không đổi.

Bài 5.

Trước hết, ta chứng minh bổ đề sau: \medskip

Bổ đề. Giá trị của biểu thức $\frac{1}{1}+\frac{1}{2}+\frac{1}{3}+\cdots+\frac{1}{n}$ tiến tới vô cực khi $n\to +\infty.$ \medskip

Thật vậy, xét hàm số $f(x)=\ln (1+x)-x$ với $x>0$. Ta có $$f'(x)=\frac{1}{1+x}-1<0$$ nên đây là hàm nghịch biến, suy ra $f(x)<f(0)=0$ hay $\ln (1+x)<x,\forall x>0$. Thay $x$ bởi $\frac{1}{n}$, ta được
$$\ln \left( 1+\frac{1}{n} \right)<\frac{1}{n}\Leftrightarrow \frac{1}{n}>\ln (1+n)-\ln n.$$ Do đó, $$\frac{1}{1}+\frac{1}{2}+\frac{1}{3}++\frac{1}{n}>\ln 2-\ln1+\ln3-\ln2+\cdots+ln(n+1)-\ln n=\ln (n+1).$$ Vì $\ln (n+1)\to +\infty $ khi $n\to +\infty $ nên $$\frac{1}{1}+\frac{1}{2}+\frac{1}{3}+\cdots+\frac{1}{n}\to +\infty.$$
Trở lại bài toán, đặt $$y_n=\frac{x_1+x_3+x_5+\cdots+x_{2n-1}}{x_2+x_4+x_6+\cdots+x_{2n}}$$ với $n\ge 1.$

Ta thấy vì $\frac{1}{n}\in \left( 0;\frac{\pi }{2} \right)$ nên $\cos \frac{1}{n}>0$, suy ra $$x_n=\frac{1}{n\cos \frac{1}{n}}>0,n\ge 1. $$
Xét hàm số $f(t)=\frac{t}{\cos t}$ với $t\in \left( 0;\frac{\pi }{2} \right)$ thì ${f}'(t)=\frac{\cos t+t\sin t}{{{\cos }^{2}}t}>0$ nên đây là hàm đồng biến. Chú ý rằng $x_n=f\left( \frac{1}{n} \right)$, mà $\frac{1}{n}$ là dãy giảm nên $x_n$ cũng là dãy giảm. \medskip

Suy ra $x_1>x_2,x_3>x_4,\ldots,x_{2n-1}>x_{2n}$ nên $y_n>1$. \medskip

Ngoài ra, ta cũng có $x_3<x_2,x_5<x_4,\ldots,x_{2n-1}<x_{2n-2}$ nên $y_n< \frac{x_1+x_2+x_4+\cdots+x_{2n-2}}{x_2+x_4+\cdots+x_{2n}}$

$= 1-\frac{x_1-x_{2n}}{x_2+x_4+\cdots+x_{2n}}<1-\frac{x_1}{x_2+x_4+\cdots+x_{2n}}$

Dễ thấy rằng $$x_2+x_4+\cdots+x_{2n}=\sum\limits_{i=1}^{n}{\frac{1}{2i\cos \frac{1}{2i}}}\ge \sum\limits_{i=1}^{n}{\frac{1}{2i}}=\frac{1}{2}\sum\limits_{i=1}^{n}{\frac{1}{i}}.$$

Theo bổ đề trên thì $\sum\limits_{i=1}^{n}{\frac{1}{i}}$ tiến tới vô cực nên $$\lim \left( x_2+x_4+\cdots+x_{2n} \right)=+\infty .$$

Do đó $$\lim \left( 1-\frac{x_{1}}{x_2+x_4+\cdots+x_{2n}} \right)=1-0=1.$$ Theo nguyên lý kẹp, ta có $\lim x_n=1.$

Bài 6.

Đặt $X=x-1,Y=y+1$, thay vào, ta có
$$\begin{aligned}
& \left\{ \begin{aligned}
& {{X}^{2}}+{{Y}^{2}}=b \\
& Y-1={{(X+1)}^{2}}+(2a+1)(X+1)+{{a}^{2}} \\
\end{aligned} \right. \
& \Leftrightarrow \left\{ \begin{aligned}
& {{X}^{2}}+{{Y}^{2}}=b \\
& Y={{X}^{2}}+(2a+3)X+{{a}^{2}}+2a+3.
\end{aligned} \right. \\
\end{aligned}$$
Ta đưa về tìm điều kiện của $b$ để tồn tại $a$ mà hệ trên có nghiệm $(X,Y).$ Do $$Y-(X+2)={{X}^{2}}+2(a+1)X+{{(a+1)}^{2}}={{\left( X+a+1 \right)}^{2}}\ge 0$$ nên $Y\ge X+2$. Suy ra $Y-X\ge 2>0$, tức là ${{(X-Y)}^{2}}\ge 4.$ Ta có
$$b={{X}^{2}}+{{Y}^{2}}=\frac{{{(X-Y)}^{2}}+{{(X+Y)}^{2}}}{2}\ge \frac{{{(Y-X)}^{2}}}{2}\ge 2.$$ Mặt khác, với $b\ge 2$, nếu chọn $X=-(a+1)$ thì có $Y=X+2=1-a$. Khi đó, ta có
$${{X}^{2}}+{{Y}^{2}}={{(a+1)}^{2}}+{{(a-1)}^{2}}=2({{a}^{2}}+1)=b.$$ Như thế, với $a$ thỏa mãn $2({{a}^{2}}+1)=b$ thì hệ có nghiệm là $$(X,Y)=(-a-1,1-a).$$ Dễ dàng thấy rằng do $b\ge 2$ nên luôn tồn tại $a$ như thế. \medskip

Vậy các giá trị cần tìm của $b$ là $b\ge 2$.

Bài 7.

(a) Giả sử ngược lại, tồn tại $2$ cặp $(A_i,B_i)$ và $(A_j,B_j)$ thỏa mãn điều kiện đề bài đã cho. \medskip

Vì $i\ne j$ nên theo giả thiết, $$\left| A_i \cap B_j \right|\ge 1,\left| A_j\cap B_i \right|\ge 1.$$ Đặt $x_r\in A_i\cap B_j,x_s\in A_j\cap B_i$ với $1\le r,s\le n$ thì:

Do $x_r\in B_j$ nên với mọi $x_k\in A_j$, ta đều có $k<r.$
Do $x_r\in A_i$ nên với mọi $x_k\in B_i$, ta đều có $k>r$.

Từ đây suy ra $$A_j \subset \left\{ x_1,x_2,\ldots,x_{r-1} \right\},B_i\subset \left\{x_{r+1},x_{r+2},\ldots,x_n \right\}.$$

Điều này cho thấy $A_j\cap B_i=\varnothing $, mâu thuẫn với giả thiết. Vậy tồn tại không quá $1$ cặp $(A_i,B_i)$ thỏa mãn điều kiện đã cho. \medskip

(b) Gọi $T$ là tập hợp các cách chọn hai dãy $$A_1,A_2,\ldots,A_m \text{và} B_1,B_2,\ldots,B_m$$ thỏa mãn điều kiện là: với mỗi $i,j\in \left\{ 1,2,3,\ldots,n \right\}$, $A_i\cap B_j=\varnothing $ nếu và chỉ nếu $i=j.$ \medskip

Gọi $T_i\subset T$ là các cách chọn sao cho sao cho cặp $(A_i,B_i)$ thỏa mãn điều kiện là: cặp $(A_i,B_i)$ với $i=1,2,3,\ldots,n$ sao cho nếu $x_k \in A_i$ và $x_l\in B_i$ thì $x_k<x_l$ (ở đây ta xét thứ tự ban đầu của các phần tử của $X$). \hfill (*) \medskip

Theo câu (a) thì $T_i \cap T_j=\varnothing $ với $i\ne j$ nên ta có $$\left| T_1 \right|+\left| T_2 \right|+\cdots +\left| T_m \right|=\left| T_1 \cup T_2 \cup \ldots \cup T_m \right|\le T.$$ Tiếp theo, với $1\le i\le m$, xét một tập hợp $S\subset X$ và $\left| S \right|=a_i+b_i$. Khi đó, tương ứng với $S$, có đúng $1$ cách chọn $(A_i,B_i)$ thỏa mãn tính chất $(*)$ – tức là $A_i$ sẽ nhận $a_i$ số nhỏ nhất trong tập $S,$ $B_i$ là lấy phần còn lại. \medskip

Trong khi đó, nếu không có điều kiện $(*),$ ta có thể chọn tùy ý $C_{a_i+b_i}^{a_i}$ phần tử trong $S$ và $A$ và số còn lại cho $B.$ \medskip

Do đó, ta có
$\left| T_i \right|=\frac{\left| T \right|}{C_{a_i+b_i}^{a_i}} $ với $i=1,2,\ldots,m.$

Từ đây suy ra

$$\sum\limits_{i=1}^{m}\frac{\left| T \right|}{C_{a_i+b_i}^{a_i}}\le \left| T \right|\Leftrightarrow \sum\limits_{i=1}^{m}\frac{1}{C_{a_i+b_i}^{a_i}}\le 1$$

Ta có đpcm.

Bài 8. 

(a) Giả sử $EF$ cắt $BC$ ở $L$ và $(T),(O)$ cắt nhau tại $J$ khác $A.$ Suy ra $AJ$ chính là trục đẳng phương của $(T),(O).$ Do đó $OT\bot AJ$. \medskip

Khi đó,
[LB\cdot LC=LE\cdot LF] nên $L$ thuộc trục đẳng phương của $(T),(O)$. Suy ra $A,J,L$ thẳng hàng. Theo định lý Brocard cho tứ giác $BEFC$ nội tiếp trong đường tròn $(I)$ thì $I$ chính là trực tâm của tam giác $ADL.$ \medskip

Vì thế nên $ID\bot AL$, mà $OT\bot AJ$ nên $ID\parallel OT$. \medskip

(b) Dễ dàng thấy rằng $D$ là trực tâm của tam giác $AGH$ nên $AD\bot GH$. Ta sẽ chứng minh rằng $A,D,K$ thẳng hàng. \medskip

Ta có $DB\cdot DF=DE\cdot DC$ nên $D$ có cùng phương tích tới $(ABF),(AEC)$. Suy ra $AD$ chính là trục đẳng phương của $2$ đường tròn này. \medskip

Bằng biến đổi các góc nội tiếp, ta thấy rằng
$$\angle MPQ=\angle MBF=\angle CEF=\angle CNQ.$$ Suy ra $MNPQ$ nội tiếp, dẫn đến $KM\cdot KP=KN\cdot KQ$, tức là $K$ cũng có cùng phương tích tới $2$ đường tròn $(ABF),(AEC)$. \medskip

Từ đó suy ra $A,D,K$ thẳng hàng. Do đó, $DK$ vuông góc với $GH.$

Đáp án thi chọn đội tuyển trường PTNK năm 2014

Đề bài

Ngày thi thứ nhất

Bài 1. Cho $a,b,c > 0$ thỏa mãn điều kiện $(a+1)(b+1)(c+1)=1+4abc$.
Chứng mình rằng ta có bất đẳng thức $a+b+c\le 1+abc.$
Bài 2. Cho tập hợp $A=\left\{ {{n}^{3}}-4n+15|n\in \mathbb{N} \right\}.$ Tìm tất cả các phần tử $a\in A$ thỏa mãn đồng thời hai điều kiện sau đây:

i) $a$ là số chẵn.
ii) Nếu $a_1,a_2$ là các ước số của $\dfrac{a}{2}$ với $a_1,a_2>1$ thì $\gcd (a_1,a_2)>1$.

Bài 3. Tìm tất cả các hàm số $f:\mathbb N^* \rightarrow \mathbb N^*$ thỏa mãn:
$$f \left(\dfrac{f(n)}{n} \right)=n^2 \ \forall n\in \mathbb N^*.$$
Bài 4. Cho tam giác $ABC$ nội tiếp $(O)$, có $B,C$ cố định và $A$ thay đổi trên $(O).$ Ký hiệu $(I)$ là đường tròn nội tiếp tam giác $ABC.$ Gọi $({{O}_{1}})$ là đường tròn qua $A,B$ và tiếp xúc với đường tròn $(I)$ tại $E.$ Gọi $({{O}_{2}})$ là đường tròn qua $A,C$ và tiếp xúc với đường tròn $(I)$ tại $F$. Đường phân giác trong của góc $\angle AEB$ cắt $({{O}_{1}})$ tại $M$ và đường phân giác trong của góc $\angle AFC$ cắt $({{O}_{2}})$ tại $N.$

a) Chứng minh rằng tứ giác $EFMN$ nội tiếp.
b) Gọi $J$ là giao điểm của $EM$ và $FN$. Chứng minh rằng đường thẳng $IJ$ luôn đi qua một điểm cố định.

Ngày thi thứ hai

Bài 5. Cho dãy số $({{x}_{n}})$ bởi $x_0=1,x_1=2014$ và $x_{n+1}=\sqrt[3]{x_nx_{n-1}^2}\ \forall n\in \mathbb{N}^*.$

a) Chứng minh rằng dãy số $(x_n)$ có giới hạn hữu hạn và tìm giới hạn đó.
b) Với mỗi $n\ge 2$, hãy tìm số nguyên dương $k$ nhỏ nhất sao cho $a=x_n^k$ là một số nguyên. Chứng minh rằng khi đó $a$ không thể viết được dưới dạng tổng các lũy thừa bậc ba của hai số tự nhiên.

Bài 6. Cho $X$ là tập hợp gồm $19$ phần tử.

a) Chứng minh rằng tồn tại ít nhất $2600$ tập con $7$ phần tử của $X$ sao cho với hai tập con $A,B$ bất kỳ trong số $2600$ tập con đó, ta có $\left| A \cap B \right|\le 5.$
b)  Xét một họ $\Omega $ gồm $k$ tập con có $7$ phần tử của $X$. Một tập $A\subset X$ được gọi là một cận trên của $\Omega $ nếu như $\left| A \right|=8$ và tồn tại một tập con $F$ của họ $\Omega $ sao cho $F\subset A$. Gọi $d$ là số tập con cận trên của họ $\Omega$. Chứng minh rằng $d\ge \frac{3}{2}k.$

Bài 7. Cho tam giác $ABC$ không cân. Gọi $I$ là trung điểm $BC$. Đường tròn $(I)$ tâm $I$ đi qua $A$ cắt $AB,AC$ lần lượt tại $M,N.$ Giả sử $MI,NI$ cắt $(I)$ tại $P,Q$. Gọi $K$ là giao điểm của $PQ$ với tiếp tuyến tại $A$ của $(I)$. Chứng minh rằng $K$ thuộc đường thẳng $BC.$
Bài 8. Tìm số nguyên dương $n$ lớn nhất thỏa mãn các điều kiện sau:

a) $n$ không chia hết cho $3$.
b) Bảng vuông $n\times n$ không thể được phủ kín bằng $1$ quân tetramino $1\times 4$ và các quân trimino $1\times 3$. Trong phép phủ, các quân tetramino và trimino được phép quay dọc nhưng không được phép chườm lên nhau hoặc nằm ra ngoài bảng vuông.

Hết

Lời giải

Bài 1.

Điều kiện đã cho viết thành $ab+bc+ca+a+b+c=3abc$.
Chia hai vế cho $abc$ rồi đặt $a=\frac{1}{x},b=\frac{1}{y},c=\frac{1}{z}$, ta có
$xy+yz+zx+x+y+z=3.$ \medskip

Bất đẳng thức đã cho có thể viết thành
$$xy+yz+zx-xyz\le 1 \text{ hay } x+y+z+xyz\ge 2. $$
Theo bất đẳng thức Schur thì $${{(x+y+z)}^{3}}+9xyz\ge 4(xy+yz+zx)(x+y+z). $$
Đặt $m=x+y+z,n=xy+yz+zx$ thì $m+n=3$ và
$$xyz\ge \frac{4mn-{{m}^{3}}}{9}. $$
Ta sẽ chứng minh rằng
$$m+\frac{4mn-{{m}^{3}}}{9}\ge 2\Leftrightarrow {{m}^{3}}+4{{m}^{2}}-21m+18\le 0 $$ hay
$(m-2)({{m}^{2}}+6m-9)\le 0.$
Chú ý rằng ${{m}^{2}}\ge 3n$ nên
$${{m}^{2}}\ge 3(3-m)\Leftrightarrow {{m}^{3}}+3m\ge 9. $$
Do đó ${{m}^{2}}+6m-9\ge 0.$
Ta xét các trường hợp

Nếu $m>2$ thì $x+y+z>2$ nên hiển nhiên bất đẳng thức cần chứng minh là đúng.
Nếu $m\le 2$ thì $m-2\le 0$ nên ta cũng có $(m-2)({{m}^{2}}+6m-9)\le 0.$

Vậy trong mọi trường hợp, ta luôn có đpcm.

Bài 2.

Ta thấy rằng $a=n^3-4n+15$ chẵn nên $n^3+15$ chẵn hay $n$ lẻ. Đặt $n=2k+1$ với $k \in \mathbb{N}$. Ta có
$\begin{aligned}
a={{n}^{3}}-4n+15 & =(n+3)({{n}^{3}}-3n+15) \\
& =(2k+4)(4{{k}^{2}}-2k+3) \\
\end{aligned} $
nên $\frac{a}{2}=(k+2)(4{{k}^{2}}-2k+3)$.
Điều kiện ii) cho thấy rằng $\frac{a}{2}$ phải là lũy thừa của một số nguyên tố, vì nếu nó có hai ước nguyên tố trở lên, đặt là $p,q$ thì chọn $x=p,y=q$, ta có $x,y>1$ nhưng $\gcd (x,y)=1,$ không thỏa. \medskip

Vì $(4{{k}^{2}}-2k+3)-(k+2)=4{{k}^{2}}-3k+1>0$ với mọi $k\in \mathbb{N}$. Do đó, ta phải có $k+2|4{{k}^{2}}-2k+3$. Suy ra
$\frac{4{{k}^{2}}-2k+3}{k+2}=4k-10+\frac{23}{k+2}\in \mathbb{Z}. $
Do đó $k+2\in \{1,23\}$ vì $k+2>0.$ Ta xét các trường hợp

Nếu $k+2=1$ thì $k=-1$ hay $n=2k+1=-1<0$, không thỏa.
Nếu $k+2=23$ thì $k=21$ hay $n=43$, tính được $\frac{a}{2}=3\cdot {{5}^{2}}\cdot {{23}^{2}}$, cũng không thỏa.

Vậy không tồn tại số $a$ nào thỏa mãn.

Bài 3.

Với $n\in \mathbb{N}*$, ta thấy rằng nếu $n=1$ thì $f(f(1))=1$. \medskip

Nếu $n>1$ thì gọi $p$ là một ước nguyên tố bất kỳ của $n$. \medskip

Vì $\frac{f(n)}{n}\in \mathbb{N}*$ nên $n|f(n)$. Đặt $a={{v}_{p}}(n),b={{v}_{p}}\left( f(n) \right)$ thì trước hết, ta có $a\le b.$ \medskip

Từ $f\left( \frac{f(n)}{n} \right)={{n}^{2}}$, ta suy ra rằng $\left. \frac{f(n)}{n} \right|{{n}^{2}}$ hay $f(n)|{{n}^{3}}$, tức là $b\le 3a.$ \medskip

Trong biểu thức đã cho, thay $n\to \frac{f(n)}{n}$ thì
$f\left( \frac{f\left( \frac{f(n)}{n} \right)}{\frac{f(n)}{n}} \right)={{\left( \frac{f(n)}{n} \right)}^{2}}\Leftrightarrow f\left( \frac{{{n}^{3}}}{f(n)} \right)={{\left( \frac{f(n)}{n} \right)}^{2}}$
Do đó, ta phải có $\left. {{\left( \frac{f(n)}{n} \right)}^{2}} \right|\frac{{{n}^{3}}}{f(n)}\Leftrightarrow \left. {{f}^{3}}(n) \right|{{n}^{5}} \text{ nên } 3b \le 5a.$$ Sau đó lại tiếp tục thay $n$ trong biểu thức đã cho bởi $\frac{{{n}^{5}}}{{{f}^{3}}(n)}$ và cứ như thế, ta xây dựng được hai dãy hệ số của $a,b$ như sau
${{u}_{0}}={{v}_{0}}=1,{{u}_{1}}=3,{{v}_{1}}=1 \text{ và } $
${{u}_{k+1}}=2{{u}_{k-1}}+{{v}_{k}},{{v}_{k+1}}=2{{v}_{k-1}}+{{u}_{k}} \text{ với } k\ge 1. $
Khi đó $\frac{{{v}_{2k}}}{{{u}_{2k}}}\le \frac{b}{a}\le \frac{{{u}_{2k+1}}}{{{v}_{2k+1}}}. $
Biến đổi công thức của hai dãy, ta có ${{u}_{n+2}}=5{{u}_{n}}-4{{u}_{n-2}},{{v}_{n+2}}=5{{v}_{n}}-4{{v}_{n-2}}$ và cả hai dãy đều có phương trình đặc trưng là ${{t}^{2}}-5t+4=0$. Ngoài ra, dãy chẵn và dãy lẻ trong mỗi dãy đều độc lập với nhau.

Ta có ${{u}_{0}}=1,{{u}_{2}}=3,{{v}_{0}}=1,{{v}_{2}}=5$ nên
\[{{u}_{2k}}=\frac{13+2\cdot {{16}^{k}}}{15},{{v}_{2k}}=\frac{11+4\cdot {{16}^{k}}}{15},k\ge 1. \] Từ đó, dễ dàng tính được $\lim \frac{{{u}_{2k+1}}}{{{v}_{2k+1}}}=2$. \medskip

Một cách tương tự, ta tính được $\lim \frac{{{u}_{2k}}}{{{v}_{2k}}}=\frac{1}{2}$. Do đó, số $\frac{b}{a}$ bị kẹp ở giữa và là số nguyên nên chỉ có thể là $\frac{b}{a}=2\Leftrightarrow b=2a.$ \medskip

Rõ ràng tập hợp ước nguyên tố của $n$ và $f(n)$ là giống nhau. Hơn nữa, với một ước nguyên tố cụ thể thì số mũ trong $f(n)$ gấp đôi số mũ trong $n.$ Suy ra $f(n)={{n}^{2}}, \forall n>1.$ \medskip

Tiếp theo, giả sử $f(1)=n>1$ thì ta có $f(f(1))=1$ nên $f(n)=1,$ mâu thuẫn. Vì thế nên chỉ có thể $f(1)=1.$ \medskip

Vậy tất cả các hàm thỏa mãn là $f(n)={{n}^{2}},\forall n\in \mathbb{N}^*$.

Bài 4.

(a) Trước hết, ta thấy rằng ${{O}_{1}},I,E$ thẳng hàng và ${{O}_{2}},I,F$ thẳng hàng. \medskip

Vì $M$ là trung điểm cung $AB$ của $({{O}_{1}})$ nên ${{O}_{1}}M$ là trung trực của $AB$, suy ra $O\in {{O}_{1}}M.$ Tương tự, ta cũng có $O\in {{O}_{1}}N.$ \medskip

Gọi $P,Q$ lần lượt là tiếp điểm của $(I)$ với $AB,AC.$ \medskip

Vì $IP\parallel {{O}_{1}}M$ (cùng vuông góc với $AB$) nên $\angle M{{O}_{1}}E=\angle PIE$.
Hơn nữa, các tam giác ${{O}_{1}}ME,IPE$ đều cân với đỉnh là ${{O}_{1}},I$ nên suy ra chúng đồng dạng, tức là
$\angle IEP=\angle {{O}_{1}}EM$ hay $E,P,M$ thẳng hàng. Tương tự thì $F,Q,N$ cũng thẳng hàng. \medskip

Vì ta đã có $E,F,P,Q$ cùng thuộc đường tròn $(I)$ nên để có $E,F,M,N$ cùng thuộc một đường tròn thì $\angle EMN=\angle EFN=\angle EPQ$ hay $MN\parallel PQ.$ \medskip

Mặt khác, $AI\bot PQ$ nên ta cần có $AI\bot MN.$ \medskip

Thật vậy, sử dụng phương tích với đường tròn $(I)$ ta có
\[M{{A}^{2}}-N{{A}^{2}}=MP\cdot ME-NQ\cdot NF=M{{I}^{2}}-N{{I}^{2}} \] nên theo định lý bốn điểm thì $AI\bot MN$, từ đó ta có đpcm. \medskip

(b) Vì $PQ\parallel MN,OM\parallel IP$ nên dễ dàng có $\angle IPQ=\angle OMN$. Tương tự $\angle IPQ=\angle ONM.$ \medskip

Do đó, hai tam giác $IPQ,OMN$ đồng dạng với nhau, tức là $$\frac{IP}{OM}=\frac{PQ}{MN}.$$
Ngoài ra, $$\frac{JP}{JM}=\frac{IP}{OM},$$ kết hợp với $\angle JPI=\angle JMO$, ta có hai tam giác $JPI,JMO$ đồng dạng, dẫn đến $$\angle PJI=\angle MJO.$$

Từ đây suy ra $I,J,O$ thẳng hàng hay $IJ$ luôn đi qua điểm $O$ cố định.

Bài 5.

(a) Đặt $u_n=\log_{2014}(x_n)$ thì ta thu được dãy $(u_n)$ như sau
$\left\{\begin{matrix} u_0=0,u_1=1\\ u_{n+1}=\dfrac{1}{3}u_n+\dfrac{2}{3}u_{n-1} \end{matrix}\right. $
Từ đó tìm được
$u_n=\dfrac{3}{5}-\dfrac{3}{5} \cdot \left ( \dfrac{-2}{3} \right )^n$
Suy ra $\underset{n\rightarrow +\infty }{\lim}u_n=\dfrac{3}{5}$ nên ta có được
$\underset{n\rightarrow +\infty }{\lim} x_n=\underset{n\rightarrow +\infty }{\lim}(2014^{u_n})=2014^{3/5} $
(b) Ta thấy rằng để có $(x_n)^k$ là một số nguyên thì $\dfrac{3k(3^{n}-(-2)^n)}{5 \cdot 3^n}\in \mathbb{Z}$ nguyên.
Ta xét các trường hợp

Nếu $n$ lẻ thì $3^n-(-2)^n=3^n+2^n\;\vdots\; 5$. Vì $\gcd\left ( \dfrac{3^n+2^n}{5},3^n \right )=1$ nên ta được $3^n\mid 3k$ nên $k$ nhỏ nhất thỏa mãn điều này là $k=3^{n-1}$.
Nếu $n$ chẵn thì $3^n-2^n\equiv (-2)^n-2^n=0 \pmod{5}$ và tương tự, ta cũng tìm được $k=3^{n-1}$.

Do đó số $k$ nhỏ nhất cần tìm là $k=3^{n-1}$.
Tiếp theo, ta sẽ chứng minh rằng phương trình sau không có nghiệm tự nhiên
$a^3+b^3=2014^n \Leftrightarrow (a+b)(a^2-ab+b^2)=2014^n $

Gọi ${{n}_{0}}$ số nguyên dương nhỏ nhất sao cho tồn tại $a,b\in {{\mathbb{Z}}^{+}}$ để ${{a}^{3}}+{{b}^{3}}={{2014}^{{{n}_{0}}}}$.
Dễ thấy ${{n}_{0}}=1$ không thỏa nên ta chỉ xét ${{n}_{0}}\ge 2.$
Ta xét các trường hợp

Nếu $\gcd (a+b,{{a}^{2}}-ab+{{b}^{2}})=1$ thì dễ thấy ${{(a-b)}^{2}}\ge 1$. Khi đó
\[{{a}^{2}}-ab+{{b}^{2}}\ge a+b>\sqrt{{{a}^{2}}-ab+{{b}^{2}}}. \] Vì $2014=2\cdot 19\cdot 53$ nên chỉ có thể xảy ra \[a+b={{19}^{{{n}_{0}}}},{{a}^{2}}-ab+{{b}^{2}}={{106}^{{{n}_{0}}}}. \] Ngoài ra ${{(a+b)}^{2}}\le 4({{a}^{2}}-ab+{{b}^{2}})$ nên ta phải có ${{361}^{{{n}_{0}}}}\le 4\cdot {{106}^{{{n}_{0}}}}$. Đánh giá này sai khi ${{n}_{0}}\ge 2$ nên trường hợp này không thỏa.
Nếu $\gcd (a+b,{{a}^{2}}-ab+{{b}^{2}})>1$ thì chẳng hạn
\[a+b={{2}^{x}}u,{{a}^{2}}-ab+{{b}^{2}}={{2}^{y}}v \text{ với } \gcd (u,2)=\gcd (v,2)=1.\] Các trường hợp còn lại chứng minh tương tự.
Ngoài ra $uv={{1007}^{{{n}_{0}}}},x+y={{n}_{0}}. $
Chú ý rằng ${{(a+b)}^{2}}-({{a}^{2}}-ab+{{b}^{2}})=3ab$ nên $3ab$ cũng chẵn, tức là cả hai số $a,b$ đều chẵn (vì nếu không thì ${{a}^{3}}+{{b}^{3}}$ lẻ).

Từ đây dễ dàng chứng minh được $3{{v}_{2}}(a)=3{{v}_{2}}(b)={{n}_{0}}$, ta đưa về
${{{x}’}^{3}}+{{{y}’}^{3}}={{1007}^{{{n}_{0}}}}$.
Cứ như thế, ta được $2014|a,2014|b$ nên phương trình sau cũng có nghiệm nguyên dương
${{\left( \frac{a}{2014} \right)}^{3}}+{{\left( \frac{b}{2014} \right)}^{3}}={{2014}^{{{n}_{0}}-3}}. $
Điều này mâu thuẫn với các chọn ${{n}_{0}}$ nên phương trình trên vô nghiệm. Các trường hợp còn lại tương tự.
\end{enumerate}
Ta có đpcm.

Bài 6.

(a) Không mất tính tổng quát, ta có thể giả sử $X$ là tập hợp $19$ số nguyên dương đầu tiên.
Gọi $X(k)$ là tập hợp tất cả các tập con có $7$ phần tử của $X$ và tổng các phần tử của nó chia $19$ dư $k$. \medskip

Khi đó, dễ thấy rằng $\left| X(0) \right|+\left| X(1) \right|+\cdots +\left| X(18) \right|$ chính là số tập con có $7$ phần tử tùy ý của $X$ và là $C_{19}^{7}.$ \medskip

Ta thấy rằng hai tập hợp $A,B\in X(k)$ tùy ý đều thỏa mãn đề bài. \medskip

Thật vậy, \medskip

Giả sử $\left| A\cap B \right|=6$ (không thể có $\left| A\cap B \right|=7$ vì khi đó hai tập hợp trùng nhau). Đặt
$A=\{{{a}_{1}},{{a}_{2}},{{a}_{3}},{{a}_{4}},{{a}_{5}},{{a}_{6}},x\},B=\{{{a}_{1}},{{a}_{2}},{{a}_{3}},{{a}_{4}},{{a}_{5}},{{a}_{6}},y\}$
thì \[\sum\limits_{i=1}^{6}{{{a}_{i}}}+x\equiv \sum\limits_{i=1}^{6}{{{a}_{i}}}+y\equiv k\pmod{19}\] nên $x\equiv y\pmod{19}$. Suy ra $x=y$, mâu thuẫn.
Đến đây, dễ thấy rằng \[\underset{0\le k\le 18}{\mathop{\max }}\,\left\{ \left| X(k) \right| \right\}\ge \frac{C_{19}^{7}}{19}=2652>2600.\]Ta có đpcm. \medskip

(b) Xét một tập hợp $F$ thuộc họ $\Omega $. Vì $\left| X\backslash F \right|=19-7=12$ nên có tất cả $12$ tập hợp $A\subset X$ với $\left| A \right|=8$ và $F\subset A.$ \medskip

Ngược lại, ứng với một tập hợp $A$ là một cận trên của họ $\Omega $, có không quá $8$ tập $F$ trong họ $\Omega $ sao cho $F\subset A.$ Do đó $d\ge \frac{12}{8}k$ hay $d\ge \frac{3}{2}k.$ \medskip

Đẳng thức xảy ra khi họ $\Omega $ là tập hợp tất cả các tập con có $7$ phần tử của $X$.

Bài 7.

Không mất tính tổng quát, giả sử $AB<AC.$ \medskip

Kẻ đường kính $AJ$ của đường tròn $(I).$ Khi đó, dễ thấy tứ giác $ABJC$ và $ANJQ$ là các hình bình hành nên $JB\parallel AC,JQ\parallel AN$ dẫn đến $J,Q,B$ thẳng hàng. Tương tự $J,P,C$ thẳng hàng. \medskip

Gọi $H$ là hình chiếu của $A$ lên $BC$ thì tứ giác $AQBH$ nội tiếp. \medskip

Suy ra
\[\angle QHB=\angle QAB=\angle QAM=\angle QPM=\angle QPI \] nên tứ giác $PQHI$ cũng nội tiếp.
Gọi $(O)$ là đường tròn ngoại tiếp tam giác $ABC$ thì dễ thấy đường tròn $(AHI)$ tiếp xúc với $(O)$ tại $A.$

Xét ba đường tròn $(O),(AHI),(PQHI)$ thì

Trục đẳng phương của $(O),(AHI)$ là tiếp tuyến của $(O)$ tại $A$.
Trục đẳng phương của $(O),(PQHI)$ là $PQ$.
Trục đẳng phương của $(PQHI),(AHI)$ là $HI.$

Do đó, $K$ chính là tâm đẳng phương của ba đường tròn nên $K\in HI$ hay $K,B,C$ thẳng hàng.

Bài 8.

Ta sẽ chứng minh $n = 5$ là giá trị lớn nhất cần tìm. \medskip

Ta nhận thấy rằng nếu $n = 3k+1, k \ge 1$ thì ta luôn phủ được bảng vuông $n \times n$ bằng cách phủ hàng đầu tiên bằng $1$ quân tetramino kích thước $1 \times 4$ (ta sẽ gọi tắt là tetramino) và $k-1$ quân trimino kích thước $1 \times 3$ (ta sẽ gọi tắt là trimino). Các cột còn lại có chiều dài $3k$ có thể phủ được bằng các quân trimino (xoay dọc lại). \medskip

Ta chứng minh rằng nếu $n = 3k+2, k \ge 2$ thì bảng vuông $n \times n$ cũng phủ được. Cách phủ với $n = 8$ được minh họa như sau

Dễ dàng thấy rằng với $k \ge 3$ thì ta có thể thu được cách phủ cho bảng vuông $n \times n$ bằng cách phủ phần hình vuông $8 \times 8$ ở góc trên bên trái như trên, phần còn lại gồm $1 $ hình chữ nhật kích thước $3(k-2) \times (3k+2)$ và 1 hình chữ nhật kích thước $8 \times 3(k-2)$ phủ được bằng các quân trimino.

Bây giờ ta chứng minh bảng vuông $5 \times 5$ không thể phủ được bằng 1 quân tetramino và 7 quân trimino.

Trước hết ta chứng minh bổ đề: Nếu bảng vuông $5 \times 5$ có thể phủ được bằng một hình vuông $1 \times 1$, ta gọi là unomino và $8$ quân trimino thì quân unomino $1 \times 1$ phải phủ ô trung tâm. \medskip

Thật vậy, \medskip

Ta đánh số các ô của bảng vuông $5 \times 5$ như hình vẽ

Ta thấy rằng một quân trimino luôn phủ đúng một ô mang số $1,$ một ô mang số $2$ và một ô mang số $3.$ Vì số các số $2$ bằng $9,$ còn số các số $1$ và $3$ bằng $8$ nên nếu phép phủ ở đề bài thực hiện được thì quân unomino phải phủ một ô mang số $2.$ \medskip

Mặt khác, ta có thể đánh số bảng vuông $5 \times 5$ bằng một cách khác

Các tính chất nói ở trên vẫn đúng cho cách đánh số này, tuy nhiên ở đây số số $1$ là $9$, còn số số $2$ và $3$ là $8.$ Do đó, một lần nữa ta kết luận quân unomino phải phủ một ô mang số $1.$ \medskip

Giao hai điều kiện cần nói trên lại, ta thấy với một cách phủ hợp lệ thì quân unomino phải phủ ô trung tâm. \medskip

Quay trở lại với vấn đề phủ bảng vuông $5 \times 5$ bằng $1$ quân tetramino và $7$ quân trimino. Nếu tồn tại một cách phủ như thế thì cắt quân tetramino thành $1$ quân unomino và $1$ quân trimino, ta thu được một phép phủ bảng vuông $5 \times 5$ bằng $1$ quân unomino và $8$ quân trimino. \medskip

Theo bổ đề thì quân unomino phải nằm ở ô trung tâm, nghĩa là một đầu của quân tetramino phải nằm ở ô trung tâm, mâu thuẫn (vì khi đó quân tetramino sẽ bị lòi ra ngoài bảng vuông). \medskip

Với những lý luận ở trên, ta kết luận $n = 5$ là giá trị lớn nhất cần tìm.

Thuật toán tham lam (Greedy Algorithms)

Thuật toán tham lam là thuật toán để giải quyết một bài toán trong đó ta chọn phương án tối ưu cho các bước địa phương, từ đó đạt được tối ưu toàn cục. Đó chưa hẳn luôn là phương án tối ưu cho bài toán nhưng thường sẽ được đạt được cực trị.

Ý tượng là trong mỗi bước, ta xét các phần tử (thuộc dạng tốt nhất, xấu nhất, lớn nhất hay nhỏ nhất) để đạt được các mục tiêu cần giải quyết.

Ví dụ ta có một xấp tiền gồm các tờ 1k, 2k, 5k, 10k, làm sao ta có thể đổi một số tiền 57k thành các tờ tiền này sao cho số tờ tiền là ít nhất.

Rõ ràng ta cứ chọn các tiền lớn nhất có thể, thuật toán là

  • Chọn lớn nhất $x \in {1, 2, 5, 10}$ thỏa $x \leq M$.
  • Tiếp tục thuật toán cho $M-x$.

Khi đó ta có

  • $x_1 = 10$, $M_1 = 47$, sau lần 1.
  • $x_2 = 10, M_2 = 37$
  • $x_3 = 10, M_3 = 27$
  • $x_4 = 10, M_4 = 17$
  • $x_5 = 10, M_5 = 7$
  • $x_6 = 5, M_6 = 2$
  • $x_7 = 2, M_7 = 0$

Vậy tập cần tìm là $\{10, 10, 10, 10, 10, 5, 2 \}$

Trên đây là một ví dụ đơn giản về thuật toán tham lam, tiếp theo ta xét một số ví dụ khác để các bạn có thể hiểu rõ hơn về thuật toán này.

Ví dụ 1. Chứng minh rằng mọi số nguyên dương $n$ thì đều có thể biểu dưới dạng một lũy thừa của 2 hoặc một tổng các lũy thừa của 2, và sự biểu diễn là duy nhất.

Lời giải
  • Nếu $n$ là lũy thừa của 2 thì coi như xong. Nếu $n$ không là lũy thừa của $2$ ta chọn một lũy thừa của 2 nhỏ hơn $n$ và gần $n$ nhất, giả sử là $2^{a_k}$. Ta có $2^{a_k} < n < 2^{a_k+1}$.
  • Tiếp theo áp dụng thuật toán cho số $n-2^k$, rõ ràng thuật toán dừng vì ta có dãy giảm và bị chặn bởi 0.
  • Khi đó ta có $n = 2^{a_1} + \cdots +2^{a_k}$.
  • Ta chứng minh sự biểu diễn này là duy nhất. Giả sử tồn tại hai cách biểu diễn $2^{a_k} + \cdots + 2^{a_1} = 2^{b_m} + \cdots 2^{b_1}$. Nếu $a_k >b_m \Rightarrow a_k \geq b_m+1$. Khi đó $a^k >a^{b_m+1}-1=\geq 2^{b_m}+2^{b_m-1} + \cdots 2^{1} + 1$
  • Do đó $VT > VP$ (vô lí)
  • Nếu $a_k = b_m$, chứng minh tương tự cũng có 2 dãy $a_1,\cdot a_k$ và $b_1,\cdots b_m$ là trùng nhau. Nên sự biểu diễn là duy nhất.

Ví dụ 2. Cho một đồ thị đơn, trong đó $d$ là bậc cao nhất. Chứng minh rằng có thể tô màu các đỉnh của đồ thị bằng $d+1$ màu sao cho hai đỉnh kề thì khác màu.

Lời giải
  • Giả sử các màu được đánh số từ 1 đến $d+1$. Ta thực hiện cách tô màu như sau
  • Chọn 1 đỉnh ta tô màu nhỏ nhất mà không xuất hiện trong các đỉnh lân cận với đỉnh đó. Việc tô màu này luôn thực hiện được vì bậc của mỗi đỉnh không hơn hơn $d$.

Ví dụ 3. Cho các tập $A_1, A_2 , \cdots, A_{2022}$ là các tập con có 3 phần tử của $X = \{1, 2, \cdots 2022 \}$. Chứng minh rằng ta có thể tô màu $674$ phần tử của $X$ sao cho với mọi tập $A_i$ có một phần tử không được tô màu.

Lời giải
  • Ta có thể làm ngược lại, tô màu các số sao cho với mỗi tập $A_i$ có ít nhất một số được tô màu, ta chứng minh là số phần tử cần tô không vượt quá $1348$ số.
  • Ta thực hiện cách tô sau: Chọn phần tử xuất hiện nhiều nhất trong các tập, tô màu phần tử đó. Lặp lại thuật toán đến khi không có tập hợp nào không có phần tử được tô màu
  • Ta chứng minh số phần tử được tô màu không vượt quá 1348 phần tử
  • Giả sử ta tô được $k$ phần tử, và các tập còn lại đều phân biệt rời nhau, có $m$ tập hợp. Rõ ràng vì mỗi bước thực hiện đều giảm được ít nhất 2 tập nên $k \leq \dfrac{2022}{2} = 1011$. (Vì cách chọn số phần từ xuất hiện trong nhiều tập nhất nên ít nhất là 2). Và các tập còn lại phân biệt rời nhau, mỗi tập có 3 phần tử nên $m \leq 1011/3 = 337$.\\
  • Khi đó ta tô màu mỗi phần tử trong $m$ tập còn lại thì sẽ thỏa đề bài, do đó số phần tử cần tô màu là $k+m \leq 1011+ 337 = 1348$.
  • Do đó ta có cách tô màu thỏa đề bài.

Ví dụ 4. Cho một bảng $2 \times n$, người ta điền vào bảng các số dương sao cho tổng hai số trong cùng một cột bằng 1. Chứng minh rằng ta có thể chọn mỗi cột một số sao cho các số trên mỗi dòng không lớn hơn $\dfrac{n+1}{4}$.

Lời giải

Ta sắp xếp từ trái sang phải theo thứ tự tăng dần các số ở hàng thứ nhất $a_1 \leq a_2 \leq \cdot \leq a_n$, hàng dưới tương ứng là $1-a_1, 1-a_2, \cdots, 1-a_n$.

Ta chọn các số ở hàng trên từ trái qua, nhiều nhất sao cho tổng $S$ không lớn hơn $\dfrac{n+1}{4}$. Ta chứng minh tổng các số ở hàng dưới trong các cột còn lại không lớn hơn $\dfrac{n+1}{4} – S$.

Giả sử chỉ số $i$ lớn nhất thỏa $a_{1}+a_{2}+\cdots+a_{i} \leq \frac{n+1}{4} .$ Ta cần chứng minh
$$
\dfrac{n+1}{4} \geq\left(1-a_{i+1}\right)+\left(1-a_{i+2}\right)+\cdots+\left(1-a_{n}\right)=(n-i)-\left(a_{i+1}+a_{i+2}+\cdots+a_{n}\right)
$$

$$
a_{i+1}+a_{i+2}+\cdots+a_{n} \geq \frac{3 n-1}{4}-i
$$
Từ $a_{1} \leq a_{2} \leq \cdots \leq a_{i+1}$, ta có
$$
\dfrac{a_{1}+a_{2}+\cdots+a_{i+1}}{i+1} \leq a_{i+1}
$$

và từ $a_{i+1} \leq a_{i+2} \leq \cdots \leq a_{n}$, ta có
$$
\dfrac{a_{i+1}+a_{i+2}+\cdots+a_{n}}{n-i} \geq a_{i+1}
$$
Từ đó ta có
$$
\dfrac{a_{i+1}+a_{i+2}+\cdots+a_{n}}{n-i} \geq \frac{a_{1}+a_{2}+\cdots+a_{i+1}}{i+1}
$$

$$
a_{i+1}+a_{i+2}+\cdots+a_{n} \geq(n-i) \cdot \frac{a_{1}+a_{2}+\cdots+a_{i+1}}{i+1}>(n-i) \cdot \frac{\frac{n+1}{4}}{i+1}
$$
do cách chọn $i$ lớn nhất.

Ta cần chứng minh $\dfrac{n-i}{i+1} \dfrac{n+1}{4} \geq \dfrac{3 n-1}{4}-i .$

hay $(n-1)^{2} \geq 4 i(n-i-1)$, áp dụng AM-GM $2 \sqrt{i(n-i-1)} \leq i+(n-i-1)=n-1$, ta có điều cần chứng minh.

Ví dụ 5. (IMOSL 2001) Một tập có 3 số nguyên phân biệt không âm $\{x, y, z\}$ thỏa $x < y< z$ được gọi là tập có tính chất P nên $\{z-y, y-z\} = \{a,b\}$ với $0 < a< b$ nguyên dương cho trước. Chứng minh rằng tập các số nguyên không âm được viết thành hợp các có tính chất P đôi một phân biệt.

Lời giải

Ta thấy có 2 trường hợp là $z=y=b, y-x=a$ hoặc $z-y=a, y-x=b$ hay tập có tính chất $P$ sẽ có một trong hai dạng {x, x+a, x+a+b} hoặc {x, x+b, x+a+b} với $x$ nguyên dương.

Với mỗi tập có tính chất $P$ là {x, y, z} ta tô màu $x$ đỏ, $y$ xanh $z$ vàng.

Ta tô màu các số nguyên dương như sau:

  • Xét số dương $k$ nhỏ nhất chưa được tô màu.
  • Nếu $k+a$ chưa được tô màu thì ta tô màu cho tập {k, k+a, k+a+b}.
  • Nếu $k+b$ chưa được tô màu thì ta tô màu cho tập {k, k+b,k+a+b}.

Ta chứng minh với cách tô màu như trên thì các tập đều phân biệt và số nào cũng được tô màu. Giả sử ta đã thực hiện tới bước thứ $n$ với phần tử $x_n$ là nhỏ nhất được tô màu. Ta tiếp tục thực hiện với kế tiếp với phần tử $x_{n+1}$ chưa được tô màu. Ta cần chứng minh các phần tử $x_{n+1}+a$ hoặc $x_{n+1}+b$ chưa được tô màu và $x_{n+1}+a+b$ chưa được tô màu trước đó.

  • Thật vậy rõ ràng là $x_{n+1}+a+b$ chưa được tô màu vì nó lớn hơn tất cả các số được tô màu. (Ở bước thứ $n$ thì $x_n$ được tô màu, nên $x_n+a+b $ là số lớn nhất được tô màu.
  • Nếu $x_{n+1} +b$ được tô màu thì không thể là màu đỏ, nếu xanh thì $x_{n+1}+b-a$ được tô màu đỏ, vô lí. Nếu màu vàng thì $x_{n+1}+b-a-b = x_{n+1}-a$ được tô màu đỏ, khi đó $x_{n+1}$ được tô màu, vô lí.

Một số bài tập rèn luyện

Bài 1. (Đức 2000) Có một số đá tổng cộng là 9 tấn cần được vận chuyển bằng xe tải. Mỗi tảng đá không nặng quá  1 tấn, mỗi xe tải chở không quá 3 tấn. Hỏi còn ít nhất bao nhiêu xe tải để chở hết số đá này cùng lúc.

Bài 2. Chứng minh rằng với mọi số nguyên dương $n$ tồn tại các số nguyên dương $a_1, a_2, \cdots, $ sao cho $a_i \leq i$ thỏa

$$n = a_1 \cdot 1! + a_2 \cdot 2! + \cdots + a_k \cdot k! $$

và sự biểu diễn này là duy nhất.

Bài 3. Gọi S là tập hợp n điểm trong mặt phẳng tọa độ. Nói rằng một cặp điểm thẳng hàng nếu hai điểm đó có cùng tọa độ x hoặc y. Chứng minh rằng S có thể được phân chia thành các tập con rời rạc sao cho

(a) mỗi tập con này là một tập hợp các điểm thẳng hàng,
và (b) tối đa $n^{3/2}$ cặp điểm phân biệt không có thứ tự trong S thẳng hàng nhưng không nằm trong cùng một tập con.

Bài 4. (IMOSL 2013). Cho $n$ là một số nguyên dương. Tìm số nguyên $k$ nhỏ nhất thỏa mãn tính chất: Cho bất kỳ số thực $a_1, · · ·, a_d$ sao cho $a_1 + a_2 + · · · + a_d = n$ và $0 \leq  a_i \leq 1$
với $i = 1, 2, · · ·, d$, có thể phân chia các số này thành $k$ nhóm (một số có thể trống)
sao cho tổng các số trong mỗi nhóm nhiều nhất là $1$.

Bài 5.  (IMO 2014). Với mỗi số nguyên dương n, Ngân hàng Cape Town phát hành tiền xu có mệnh giá $\dfrac{1}{n}$. Đưa ra một bộ sưu tập hữu hạn các đồng tiền như vậy (không nhất thiết phải có mệnh giá khác nhau) với tổng số giá trị nhiều nhất là $99+ \dfrac{1}{2}$. Chứng minh rằng có thể chia bộ sưu tập này thành 100 nhóm hoặc ít hơn, chẳng hạn như mà mỗi nhóm có tổng giá trị nhiều nhất là 1.

Đáp án thi chọn Đội Tuyển Trường PTNK năm học 2013-2014

Đề thi và đáp án kì thi chọn đội tuyển Toán trường Phổ thông Năng khiếu – ĐHQG TPHCM được tổ chức vào tháng 10 năm 2013, chọn ra 6 học sinh dự thi kì thi HSG Quốc gia năm 2014. Các thí sinh từ các lớp 11, 12 (chủ yếu là học sinh chuyên toán), thực hiện bài thi trong 2 ngày, mỗi ngày 4 bài, mỗi bài 180 phút. Sau đây là đề thi và đáp án thực hiện bởi Star Education.

Ngày thi thứ 1

Bài 1. Tìm tất cả các hàm số $f: \mathbb{R} \rightarrow \mathbb{R}$ thoả mãn

$$f(x^{3}+y+f(y))=2 y+x^{2} f(x), \forall x, y \in \mathbb{R}$$

Bài 2. Cho dãy $\left\{u_{n}\right\}$ thoả mãn $u_{1}=2013, u_{n+1}=u_{n}^{3}-4 u_{n}^{2}+5 u_{n} \forall n \in \mathbb{N}^{*}$. Tìm tất cả các số nguyên tố $p$ là ước của $\left(u_{2014}+2009\right)$ và $p \equiv 3(\bmod 4)$.

Bài 3. Trong một hội nghị khoa học có 5000 đại biểu tham dự, mỗi một đại biểu biết ít nhất một thứ tiếng. Một uỷ ban gồm một số đại biểu được gọi là “uỷ ban làm viẹc” nếu tất cả thành viên trong uỷ ban đều biết chung một thú tiếng; gọi là “uỷ ban thách thức” nếu không có hai thành viên nào của uỷ ban biết chung một thứ tiếng (uỷ ban có thểgồm 1 thành viên; uỷ ban này gọi là làm việc họ̆c thách thức đều được). Chứng minh rằng có thể chia các đại biểu thành 100 uỷ ban rời nhau (mỗi đại biểu thuộc một uỷ ban) sao cho các uỷ ban này họ̆c là uỷ ban làm việc hoặc là uỷ ban thách thức.

Bài 4. Tam giác $A B C$ có $B, C$ cố định còn $A$ di động sao cho $A B=A C$ và $\angle B A C>60^{\circ} .$ Đường thẳng đối xúng với $B C$ qua $A B$ cắt AC tai $P$. Trên đoạn $P C$ lấy $M$ sao cho $P M=P B$. Gọi $N$ là giao điểm của $A B$ với phân giác ngoài góc BCA. Chứng minh $M N$ luôn đi qua một điểm cố định.

Ngày thi thứ 2

Bài 5. Cho 2014 số thực $x_{1}, x_{2}, \ldots, x_{2014}$ thỏa mãn điều kiện $\sum_{i=1}^{2014} x_{i}=0$ và $\sum_{i=1}^{2014} x_{i}^{2}=2014$. Tìm giá trị lớn nhất của biểu thức $P=x_{1} x_{2} \cdots x_{2014}$.

Bài 6. Cho dãy số $u_{n}$ xác định bởi $u_{1}=1, u_{n+1}=\frac{u_{n}}{\sqrt{u_{n}^{2}+1}+\sqrt{2}}$ với mọi $n \in \mathbb{N}^{*}$. Tìm giới hạn $\lim \frac{u_{n+1}}{u_{n}}$.

Bài 7. Cho n nguyên dương và A là tập con khác rỗng của $X={1,2, \ldots, n}$.

  1. Tính giá trị của tổng $S(A)=\sum_{E C X} \cdot(-1)^{|E \cap A|}$,trong đó $E$ lấy trên tất cả các tập con của tập $X$ (kể cả tập rỗng).

  2. Cho $m \in \mathbb{N}^{*}$,xét $m$ tập con khác rỗng của $X$ là $A_{1}, A_{2}, \ldots, A_{m}$ và $m$ số nguyên khác không là $a_{1}, a_{2}, \ldots, a_{m}$ sao cho $a_{1}+a_{2}+\cdots+a_{m}<0$. Chứng minh tồn tại tập con $E$ của $X$ sao cho $\sum_{i=1}^{m}(-1)^{|E \cap A|} a_{i}>0$ (Kí hiệu $|A|$ chỉ số phần tử của tập $A$, số phần tử của tập rỗng là 0 ).

Bài 8. Cho tam giác $A B C$ nhọn có $H$ là trực tâm và $P$ là điểm di động bên trong tam giác $A B C$ sao cho $\angle B P C=\angle B H C$. Đường thẳng qua $B$ và vuông góc với $A B$ cắtPC tại $M$.Đường thẳng qua $C$ và vuông góc với $A C$ cắt $P B$ tại N. Chứng minh rằng trung điểm I của $M N$ luôn thuộc một đường cố định.

Hết

Giải

Bài 1.

Trong phương trình đã cho, thay $x=y=0$, ta có $f(f(0))=0$. \medskip

Lại thay $y=0$ thì $$f(f^3+f(0))=x^2f(x), \, \forall x.$$

Thay $y=f(0)$ thì $$f(x^3+f(0))=2f(0)+x^2f(x).$$

Từ đây suy ra $f(0)=0$. Thay $y=0$ vào đẳng thức đã cho ta được $f(x^3)=x^2f(x)$. Do đó ta có $$f(x^3+y+f(y))=2y+f(x^3) \text{ hay } f(x+y+f(y))=2y+f(x). \eqno{(*)}$$
Thay $y$ bởi $-y$, ta được $$f(x-y+f(-y))=-2y+f(x).$$
Với $x$ bất kì, ta lấy $2y=f(x)$ ta được $f(x-y+f(-y))=0$ suy ra $x-y+f(-y)=0$. Do đó, ta được $f(-x)=f(-y+f(-y))=-2y=-f(x).$
Từ đây suy ra
$$f(x+f(y)+f(f(y)))=2f(y)+f(x).$$
Trong $(*)$ thay $x=-y$ ta được $f(f(y))=2y+f(-y)=2y-f(y)$, kết hợp với đẳng thức trên, ta được $$f(x+2y)=2f(y)+f(x).$$ Đến đây cho $x=0$ ta được $f(2y)=2f(y)$ nên ta được $f(x+y)=f(x)+f(y)$, tức là $f(x)$ cộng tính.
Đến đây ta sẽ tính $f((x+1)^3+(x-1)^3)$ theo hai cách như sau

  • $f((x+1)^3+(x-1)^3)=f(2x^3+6x)=2x^2f(x)+6f(x).$
  • $f((x+1)^3+(x-1)^3)=(x+1)^2f(x+1)+(x-1)^2f(x-1)=(x+1)^2(f(x)+f(1))+(x-1)^2(f(x)-f(1))=2x^2f(x)+2f(x)+4xf(1).$

So sánh hai đẳng thức trên, ta được $f(x)=xf(1)=ax$ với mọi $x$. Thử lại ta được $a=1, a=-2$. \medskip

Vậy các hàm cần tìm là $f(x)=x, f(x)=-2x$.

Bài 2.

Ta có
$$\begin{aligned} u_{n+1}-2 & =(u_n-2)(u_{n-1}-1)^2 \\
& = (u_{n-2}-1)^2(u_{n-1}-1)^2(u_{n-2}-2) \\
&= (u_{n-1}-1)^2(u_{n-2}-1)^2 \cdots (u_2-1)^2(u_1-2). \end{aligned} $$

Do đó $$u_{2014}+2009= 2011 \left[ (u_{2013}-1)^2(u_{2012}-1)^2 \cdots (u_2-1)^2 +1 \right].$$

Gọi $B$ là biểu thức trong dấu ngoặc vuông thứ hai. Ta có bổ đề quen thuộc là nếu $a^2+b^2$ chia hết cho số nguyên tố $p=4k+3$ thì $a,b$ cùng chia hết cho $p.$ Từ đây suy ra số $B$ có dạng $a^2+1$ nên nó sẽ không có ước nguyên tố dạng $4k+3$. \medskip

Vậy $u_{2014}+9$ chỉ có một ước nguyên tố $p \equiv 3 \pmod{4}$ duy nhất là $2011$.

Bài 3. Trước hết, ta chứng minh bổ đề sau \medskip

Định lý Ramsey Với $s,t$ là các số nguyên dương, gọi $R(s,t)$ là số đỉnh ít nhất cần có của một graph để trong đó luôn tồn tại một tập độc lập $s$ đỉnh hoặc một graph con đầy đủ $t$ đỉnh. Khi đó
$$R(s,t)\le C_{s+t-2}^{s-1}. \eqno{(*)}$$

Chứng minh
Ta sẽ chứng minh rằng $$R(s,t)\le R(s-1,t)+R(s,t-1).$$
Để ý rằng với $s=1$ hoặc $t=1$ thì $R(s,t)=1$. Do đó, nếu chứng minh được đánh giá này thì chỉ cần dùng tính chất của tam giác Pascal để có $$R(s,t)\le C_{s+t-3}^{s-2}+C_{s+t-3}^{s-1}=C_{s+t-2}^{s-1}.$$
Đặt $n$ là vế phải của (*) và xét graph $G$ có $n$ đỉnh. Xét $v\in G$ thì

  • Nếu như có ít nhất $R(s,t-1)$ đỉnh kề với $v$. Khi đó, theo định nghĩa thì trong tập đỉnh đó, sẽ luôn có một tập độc lập $s$ đỉnh hoặc một graph con đầy đủ $t-1$ đỉnh, ghép thêm đỉnh $v$ vào thì thỏa mãn điều kiện của $R(s,t).$
  • Nếu như có ít nhất $R(s-1,t)$ đỉnh không kề với $v$. Tương tự trên, trong tập đỉnh đó, cũng sẽ có một một graph con đầy đủ $t$ đỉnh hoặc tập độc lập $s-1$ đỉnh, ghép thêm đỉnh $v$ vào thì thỏa mãn điều kiện của $R(s,t).$

    Từ đó, ta thấy graph $G$ này thỏa mãn điều kiện của $R(s,t)$ nên theo tính nhỏ nhất thì $R(s,t)\le n.$

Trở lại bài toán, \medskip

Xét graph đơn vô hướng $G=(V,E)$ đại diện cho hội nghị khoa học đã nêu, trong đó $V$ là tập hợp các đại biểu và hai đỉnh được nối nhau nếu hai đại biểu tương ứng quen nhau. Ta gọi $T$ là tập hợp đỉnh biểu diễn cho thành viên của ban tổ chức. \medskip

Khi đó một ủy ban gồm $5$ thành viên là đại diện nếu như đó là một graph đầy đủ, còn đó là thách thức nếu đó là graph không có cạnh. Ta gọi các graph con như thế là graph con “chuẩn”. \medskip

Trong các đỉnh $V\backslash T,$ ta xóa dần dần các graph con chuẩn đến khi không thực hiện được nữa. Ta gọi tập hợp còn lại là $S.$ Ta sẽ chứng minh rằng $S\cup T$ có thể phân hoạch thành các graph con chuẩn như trên. \medskip

Theo định lý Ramsey, rõ ràng $|S| \le C_{8}^{4}=70$. Xét một đỉnh $v \in S$ thì giả thiết, $v$ kề với cả $280$ đỉnh của $T$ nên ta chọn ra trong đó $4$ đỉnh để ghép với $v$ tạo thành một graph con “chuẩn”. Cứ như thế thực hiện cho đến hết các phần tử của $S$, còn lại bao nhiêu phần tử trong $T$ thì chia đều ra thành các graph con “chuẩn” là được. \medskip

Bài toán được giải quyết.

Bài 4.

Tam giác $PBM$ cân tại $P$ nên bằng biến đổi góc, ta có

$$\angle{PBM}=\angle{PMB} \Rightarrow 2\angle{ABC}-\angle{MBC}= \angle{ACB}+\angle{MBC}.$$

Do đó $\angle{ABC}=2\angle{MBC}$ nên $BM$ là tia phân giác của $\angle{ABC}.$ Theo tính chất đường phân giác thì
$$\frac{MC}{MA}=\frac{BC}{BA}=\frac{BC}{AC}.$$

Lại có $CN$ là phân giác ngoài của $\angle{ACB}$ nên ta cũng có
$\frac{NA}{NB}=\frac{CA}{CB}.$ Gọi $I$ là trung điểm của $BC$ thì $I$ là điểm cố định. \medskip

Xét tam giác $ABC$ với $I$ thuộc $BC$ , $M$ thuộc $AC$ và $N$ thuộc $AB$ với

$$\frac{IB}{IC} \cdot \frac{MC}{MA} \cdot \frac{NA}{NB}=1 \cdot \frac{BC}{AC} \cdot \frac{AC}{BC}=1$$

thì theo định lý Menelaus đảo, ta có $M , N , I$ thẳng hàng. \medskip

Vậy $MN$ luôn đi qua điểm $I$ cố định.

Bài 5. 

Rõ ràng có thể chọn giá trị các biến thích hợp để $P>0$ nên để tìm giá trị lớn nhất của $P$ thì ta chỉ xét các số ${{x}_{1}},{{x}_{2}},\ldots ,{{x}_{2014}}$ đều khác $0$ và số các số âm là chẵn. Không mất tính tổng quát, giả sử
${{x}_{1}}\ge {{x}_{2}}\ge \ldots \ge {{x}_{2m}}>0>{{x}_{2m+1}}\ge \ldots \ge {{x}_{2014}}.$
Đổi dấu các số ${{y}_{k}}=-{{x}_{k}}>0$ với $2m+1\le k\le 2014.$ Khi đó ta viết lại
$$\left\{ \begin{aligned}
& {{x}_{1}}+{{x}_{2}}+\cdots +{{x}_{2m}}={{y}_{1}}+{{y}_{2}}+\cdots +{{y}_{2n}}=A \\
& x_{1}^{2}+x_{2}^{2}+\cdots +x_{2m}^{2}+y_{1}^{2}+y_{2}^{2}+\cdots +y_{2n}^{2}=2014 \\
\end{aligned} \right.$$
trong đó $m+n=1007$ (ngoài ra $m,n>0$ vì các số đã cho không thể toàn bộ là dương hoặc toàn bộ là âm). Theo bất đẳng thức Cauchy-Schwarz thì
$$2014\ge \frac{{{A}^{2}}}{2m}+\frac{{{A}^{2}}}{2n} \text{ nên } {{A}^{2}}\le 4mn.$$
Lại theo bất đẳng thức AM-GM thì
$$\begin{aligned} P& =({{x}_{1}}{{x}_{2}}\ldots {{x}_{2m}})({{y}_{1}}{{y}_{2}}\ldots {{y}_{2n}})\le {{\left( \frac{A}{2m} \right)}^{2m}}{{\left( \frac{A}{2n} \right)}^{2n}} \\
&=\frac{{{A}^{2m+2n}}}{{{2}^{2m+2n}}{{m}^{2m}}{{n}^{2n}}}\le \frac{{{(4mn)}^{m+n}}}{{{2}^{2m+2n}}{{m}^{2m}}{{n}^{2n}}}={{\left( \frac{m}{n} \right)}^{n-m}}. \end{aligned}$$

Do $m,n$ khác tính chẵn lẻ nên với vai trò bình đẳng của $m,n,$ ta có thể giả sử $m<n$ nên $n-m\ge 1$ và $m\le 503.$ Khi đó, áp dụng bất đẳng thức Bernoulli thì

$${{\left( \frac{n}{m} \right)}^{n-m}}\ge 1+\left( \frac{n}{m}-1 \right)(n-m)=1+\frac{{{(n-m)}^{2}}}{m}\ge 1+\frac{1}{503}=\frac{504}{503}.$$
Suy ra $P\le {{\left( \frac{m}{n} \right)}^{n-m}}\le \frac{503}{504}.$ Đây chính là giá trị lớn nhất cần tìm, dấu bằng xảy ra khi
$m=503,n=504$ và $${{x}_{1}}={{x}_{2}}=\cdots ={{x}_{1006}}=\sqrt{\frac{504}{503}},{{x}_{1007}}={{x}_{1008}}=\cdots ={{x}_{2014}}=-\sqrt{\frac{503}{504}}.$$

Bài 6.

Xét hàm số $f(x)=\frac{x}{\sqrt{{{x}^{2}}+1}+\sqrt{2}}$ với $x\in \mathbb{R}$ thì $${f}'(x)=\frac{1+\sqrt{2+2{{x}^{2}}}}{\sqrt{1+{{x}^{2}}}{{\left( \sqrt{2}+\sqrt{1+{{x}^{2}}} \right)}^{2}}}>0$$ nên hàm này đồng biến trên $\mathbb{R}.$
Dãy số đã cho được viết lại thành
$$\left\{ \begin{aligned}
& {{u}_{1}}=1, \\
& {{u}_{n+1}}=f({{u}_{n}}),n\ge 1 \\
\end{aligned} \right.$$ thì ${{u}_{1}}<{{u}_{2}}$ nên dễ dàng chứng minh quy nạp được rằng dãy này giảm. \medskip

Do dãy này bị chặn dưới bởi $0$ nên nó có giới hạn, đặt giới hạn đó là $L\ge 0$. Trong biểu thức xác định dãy, cho $n\to +\infty ,$ ta được $$L=\frac{L}{\sqrt{{{L}^{2}}+1}+\sqrt{2}}$$ nên $L=0.$
Từ đó suy ra
$$\underset{n\to +\infty }{\mathop{\lim }}\,\frac{{{u}_{n+1}}}{{{u}_{n}}}=\underset{n\to +\infty }{\mathop{\lim }}\,\frac{1}{\sqrt{u_{n}^{2}+1}+\sqrt{2}}=\frac{1}{1+\sqrt{2}}.$$

Bài 7.

(a) Nếu $A=X$ thì $$S(A)=\sum\limits_{E\subset X}(-1)^{|E|}=C_n^0-C_n^1+C_n^2-\cdots +(-1)^nC_n^n=0.$$

Còn nếu $A\neq X$, do $S(A)$ chỉ phụ thuộc vào số phần tử của $A$ nên không mất tính tổng quát, ta giả sử rằng $A=\{1,2,\ldots ,k\}$ với $k<n$. Khi đó, ta có
$$\begin{aligned} S(A) & =\sum_{E\subset X-\{k\}}(-1)^{|E\cap A|}+\sum_{E\subset X-\{k\}}(-1)^{|(E\cup\{k\})\cap A|} \\
& =\sum_{E\subset X-\{k\}}(-1)^{|E\cap A|}-\sum_{E\subset X-\{k\}}(-1)^{|E\cap A|}=0. \end{aligned} $$
Vậy $S(A)=0,\forall A\subset X$. \medskip

(b) Đặt $f(E)=\sum_{i=1}^{m}(-1)^{|E\bigcap A_i|}a_i$. Giả sử $f(E)\leq 0, \, \forall E$. Mà ta cũng có
$$\sum_{E\subset X}f(E)=\sum_{i=1}^ma_iS(A_i)=0.$$
Suy ra $f(E)=0,\, \forall E \subset X$, nhưng điều này là không thể vì $f(\varnothing)<0$. Vậy luôn tồn tại $E$ sao cho $f(E)>0$.

 

Bài 8. 

Vẽ đường kính $AA’$ của đường tròn $(ABC)$. Vì $A’B \perp AB$ nên $B,A’,M$ thẳng hàng. Tương tự thì $C,A’,N$ thẳng hàng. Giả sử
$BA’, CA’$ cắt lại $(BHC)$ lần lượt tại $E,F$. Mặt khác

$$\angle NPM=180{}^\circ -\angle BHC=\angle A=180{}^\circ -\angle B{A}’C=\angle M{A}’N$$

nên $PA’MN$ là tứ giác nội tiếp.

Ta sẽ chứng minh trung điểm của $A’F, A’E, MN$ là thẳng hàng. Theo định lý Menelaus đảo thì điều nào tương đương với $$ \dfrac{\overline{A’F}}{\overline{A’N}} = \dfrac{\overline{EA’}}{\overline{EM}} \Leftrightarrow \dfrac{\overline{A’F}}{\overline{A’E}} = – \dfrac{\overline{A’N}}{\overline{EM}} \Leftrightarrow \dfrac{A’B}{A’C} = \dfrac{A’N}{ME}. \eqno{(*)}$$

Vì $\angle BN{A}’=\angle CME$ và $\angle NB{A}’=\angle MCE$ nên hai tam giác $BN{A}’,CME$ đồng dạng với nhau. Do đó
$\frac{{A}’N}{ME}=\frac{{A}’B}{CE}$.
Mặt khác, bằng biến đổi góc, ta cũng có $C{A}’E$ cân tại $C$ nên $CE=C{A}’.$ Ta có được $$\frac{{A}’N}{ME}=\frac{{A}’B}{{A}’C}.$$
Do đó, khẳng định $(*)$ là đúng. Vậy nên điểm $I$ luôn nằm trên đường trung bình của tam giác $A’EF$ là đường cố định.

Bạn đọc có thể tìm thêm nhiều cách giải cho bài 8 này tại

link sau

Tham khảo từ sách “Tuyển tập đề thi môn Toán đội tuyển và dự tuyển trường PTNK”

Đáp án thi chọn Đội Tuyển PTNK năm học 2012-2013

Kì thi chọn đội tuyển thi HSG Quốc gia của trường PTNK năm học 2012 – 2013 được diễn ra trong hai ngày, mỗi ngày 4 bài làm trong 180 phút.

Ngày thi thứ 1

Bài 1. Giải hệ phương trình:

$$ \begin{cases}x+y & =3 \\ x z+y t & =5 \\ x z^{2}+y t^{2} & =41 \\ x z^{3}+y t^{3} & =121\end{cases} $$

Bài 2.Cho dãy $\left(u_{n}\right)$ giảm và có giới hạn bằng 0 . Xét hai dãy số sau

$$ \begin{gathered} v_{n}=u_{1}+u_{2}+\cdots+u_{n}-n u_{n+1} \ z_{n}=u_{1}+u_{2}+\cdots+u_{n} \end{gathered} $$

Chứng minh nếu $\left(v_{n}\right)$ bị chặn thì $\left(z_{n}\right)$ hội tụ.

Bài 3. Cho tập $X={1 ; 2 ; 3 ; \ldots ; 4 n} .$ Hai tập con $A$ và $B$ của $X$ được gọi là không giống nhau nếu $|A \Delta B| \geq 2 n+1$. Trong đó $A \Delta B=(A \backslash B) \cup(B \backslash A)$. Cho $\left\{A_{1} ; A_{2} ; \ldots ; A_{m}\right\}$ là tạp con của $X$ gồm $m$ phần tử đôi một không giống nhau.

  1. Chứng minh $m \leq 2 n$.
  2. Chứng minh $m \leq \frac{4(n+1)}{3}$.

Bài 4. Cho $\triangle A B C$ với $M, N$ thuộc cạnh $B C$ sao cho $\angle B A M=\angle C A N=\alpha$ (M nằm giữa $B, N)$. Gọi I là trung điểm BC. Kẻ BH $\perp A M, C K \perp A N$ lần lượt tại $H, K$.

  1. Chứng minh tâm đường tròn (IHK) luôn thuộc một đường thẳng cố định.
  2. Tính $\alpha$ theo $\angle A B C$ và $\angle A C B$ sao cho $(I K H)$ tiếp xúc với đường tròn đường kính $\mathrm{AB}$ hoặc đường tròn đường kính $\mathrm{AC}$.

Ngày thi thứ 2

Bài 5.

  1. Chứng minh rằng với mọi $x>0$ thi $2\left(x^{\frac{4}{3}}+\frac{1}{x^{\frac{4}{3}}}+1\right) \geq 3\left(x+\frac{1}{x}\right)$.
  2. Tim số thực dương a nhỏ nhất sao cho $2\left(x^{a}+\frac{1}{x^{a}}+1\right) \geq 3\left(x+\frac{1}{x}\right)$ với mọi $x>0$

Bài 6. Tìm $n$ tự nhiên sao cho $A_{n}=1+3^{20\left(n^{2}+n+1\right)}+9^{14\left(n^{2}+n+1\right)}$ là số nguyên tố.

Bài 7. Tìm hàm số $f: N * \rightarrow N$ * thỏa mãn đồng thời hai điều kiện sau:

i) $f(m f(n))=n^{6} \cdot f(m n)$

ii) Với mọi $m, n$ là các số nguyên tố cùng nhau thì $f(m), f(n)$ cũng nguyên tố cùng nhau.

Bài 8. Cho tam giác $A B C$ có $A B=A C$. Gọi $H$ là chân đường vuông góc hạ từ A xuống $B C$. ( $C)$ là một đường tròn đi qua $H$ có tâm $I$ di động trên đoạn $A H .(C)$ cắt $A B$ tại $M, N$ và cắt $A C$ tai $P, Q$ sao cho $M$ nằm giữa $A$ và $N, P$ nằm giũa $A v \dot{a} Q$. $B P$ và $B Q$ cắt $(C)$ tại $E, F$. Chúng minh rằng giao điểm $D$ của $N E$ và $M F$ là một điểm cố định.

Giải

Ngày thi thứ 1

Bài 1.

Gọi 4 phương trình của hệ lần lượt là $(1), (2), (3), (4).$ Nếu $z+t=0$ thì khử $t$ ở các phương trình.

Ta có $x+y=3$ và $(x+y){{z}^{2}}=41$ nên ${{z}^{2}}=\frac{41}{3}$.

Mặt khác $z(x-y)=5$ và ${{z}^{3}}(x-y)=101$ nên ${{z}^{2}}=\frac{101}{5},$ từ đây ta có điều vô lý.

Do đó, ta chỉ cần xét $z+t \neq 0$. Từ $(2),$ ta có

$$(xz + yt)(z + t) = 5(z + t),$$

sau khi khai triển, sử dụng $(1),(3)$, ta được $$41 + 3zt = 5(z + t).$$

Từ $(3),$ ta có $$(x{z^2} + y{t^2})(z + t) = 41(z + t),$$

lại khai triển, sử dụng $(2),(4)$, ta được $$101 + 5zt = 41(z + t).$$

Do đó, ta có một hệ phương trình theo biến $zt,z+t$ nên giải ra được $z+t=1,zt=-12$. Từ đây ta có $(z,t)=(4,-3),(-3,4).$

  •  Nếu $z=4,t=-3$ thì dễ dàng tính được $x=2,y=1.$
  •  Nếu $z=-3,t=4$ thì ta cũng tính được $x=1,y=2.$

Vậy hệ phương trình có hai nghiệm phân biệt là $$(x,y,z,t)=(1,2,-3,4),(2,1,4,-3).$$

Bài 2.

Do $z_n$ là dãy tăng nên ta chỉ cần chứng minh nó bị chặn trên là được.

Giả sử $L$ là chặn trên của $v_n$, ta có do $x_n$ giảm nên

$${z_n} – n{x_{n + 2}} \le {v_{n + 1}} = {x_1} + {x_2} + \ldots + {x_n} + {x_{n + 1}} – (n + 1){x_{n + 2}} \le L.$$

Suy ra \[{z_n} \le L + n{x_{n + 2}}.\]

Tương tự, \[{z_n} – n{x_{n + 3}} \le {v_{n + 2}} = {x_1} + {x_2} + \ldots + {x_n} + {x_{n + 1}} + {x_{n + 2}} – (n + 2){x_{n + 3}} \le L\]

nên ta được \[{z_n} \le L + n{x_{n + 3}}.\]

Qua hai bước trên, làm tương tự, cho $n$ cố định, ta suy ra được\[{z_n} \le L + n\mathop {\lim }\limits_{N \to \infty } {x_N} = L.\]

Ta có điều phải chứng minh.

Bài 3.

(a) Ta giải bằng phương pháp đếm bằng hai cách. Giả sử ngược lại, tồn tại tập $M$ gồm $2n+1$ tập con đôi một không giống nhau. Ta đếm số các bộ $(x, i, j)$ với $i < j$ mà $x \in A_i \Delta A_j.$ Theo giả thiết, số bộ này ít nhất là

$(2n+1)\frac{(2n+1)2n}{2}.$

Với mỗi $x,$ giả sử có $k$ tập con $A_i$ chứa $x$ và $2n+1-k$ tập con không chứa $x.$ Khi đó số cặp $A_i, A_j$ mà $x \in A_i \Delta A_j$ sẽ không quá $k(2n+1-k) \le n(n+1).$ Như vậy số bộ này nhiều nhất là $4n \cdot n(n+1).$ Từ đó ta suy ra bất đẳng thức   \[(2n+1)\frac{(2n+1)2n}{2}\le 4n \cdot n(n+1).\]

Suy ra $1 ≤ 0,$ vô lý, kéo theo $m \le 2n.$

(b) Xét tập $M = \{A_1, A_2, \ldots, A_m \}$ gồm $m$ tập con đôi một không giống nhau của $X.$

Đặt $X’ = X \cup {0}.$ Ta thiết lập các tập $A’_i$ theo nguyên tắc sau đây

  •  Nếu $|A_i|$ chẵn thì $A’_i = A_i,$
  •  Nếu $|A_i|$ lẻ thì $A’_i = A_i \cup \{0\}.$

Khi đó ta có $|A’_i|$ chẵn với mọi $i.$ Từ đó, do

$$| A \Delta B | = | A | + | B | – 2| A \cap B | \text{ nên } |A’_i \Delta A’_j|.$$

Mà ${{{A}’}_{i}}\Delta {{{A}’}_{j}} \supseteq {{A}_{i}}\Delta {{A}_{j}}$ nên $\left| {{{{A}’}}_{i}}\Delta {{{{A}’}}_{j}} \right|\ge \left| {{A}_{i}}\Delta {{A}_{j}} \right|\ge 2n+1.$ Từ đây, do tính chẵn, suy ra $|A’_i \Delta A’_j| \ge 2n+2.$

Lại áp dụng lập luận tương tự (a) cho tập $M’ = \{A’_1, A’_2, \ldots, A’_m \}$ thuộc $X’,$ ta có số bộ $(x, i, j)$ với $x \in A’_i \Delta A’_j$ một mặt sẽ có ít nhất là $$(2n+2)\frac{m(m-1)}{2}.$$ Mặt khác sẽ không vượt quá $(4n+1) \cdot \frac{{{m}^{2}}}{4}.$ Từ đó suy ra bất đẳng thức

\[\begin{aligned} & (2n+2)\frac{m(m-1)}{2}\le (4n+1) \cdot \frac{{{m}^{2}}}{4} \\ &\Leftrightarrow 4(n+1)(m-1)\le (4n+1)m \\ & \Leftrightarrow m\le \frac{4(n+1)}{3}. \end{aligned}\]

Bài 4.

(a) Gọi $AT$ là đường cao của tam giác $ABC$ với $T \in BC.$ Suy ra tứ giác $AHTB$ và $ATKC$ nội tiếp.

Không có mô tả.

Do đó $$\angle{HTI}=\angle{BAH}=\alpha =\angle{CAK}=\angle{ITK}.$$ Suy ra $I,H,T,K$ đồng viên. Do đó, tâm $(IHK)$ thuộc trung trực của $TI$ cố định.

(b) Gọi $(\omega)$ là đường tròn đường kính $AB.$ Ta có tứ giác $AHTB$ nội tiếp $(\omega).$ Do đó $(\omega)$ giao $(IHK)$ tại $T$ và $H.$

Giả sử $(\omega)$ và $(IHK)$ tiếp xúc thì $T \equiv H$ dẫn đến $AM$ là đường cao của tam giác $ABC.$

Khi đó ta tính được $\alpha = 90^{\circ} -\angle{ABC}.$

Hoàn toàn tương tự nếu $(IHK)$ tiếp xúc với đường tròn đường kính $AC.$

Ngày thi thứ 2

Bài 5.

(a) Đặt $x^{\frac{4}{3}} = a, x^{-\frac{4}{3}}=b$ thì ta có $ab=1$ và cần chứng minh rằng

$$2(a^4+b^4+1) \ge 3(a^3+b^3).$$

Ta cũng có

$a^4+b^4=((a+b)^2-2)^2-2$ và $a^3+b^3=(a+b)^3-3(a+b)$.

Đặt $t = a+b \ge 2$ thì ta cần có

$2((t^2-2)^2-1) \ge 3(t^3-3t)$.

Biến đổi tương đương bất đẳng thức này, ta được

$$(t-2)(2t+1)(t^2-3) \ge 0.$$

Bất đẳng thức này đúng do $t \ge 2$.

(b) Ta chứng minh hằng số nhỏ nhất là $\alpha_0=\sqrt{3/2}.$ Không mất tính tổng quát, giả sử $x\ge 1.$ Đầu tiên, vì hàm $h(\alpha)=x^{\alpha}+\frac{1}{x^{\alpha}}$ tăng nên ta dễ thấy $\alpha>1.$ Bây giờ ta sẽ chứng minh bằng hai bước. \medskip

Bước : $\alpha=\sqrt{3/2}=\alpha_0$ thì bất đẳng thức đúng. Thật vậy, ta cần chứng minh

$$\frac{2(x^{\alpha_0}-1)^2}{x^{\alpha_0}}\ge\frac{3(x-1)^2}{x},$$

tương đương $f(x)=x^{\alpha_0}-\frac{1}{x^{\alpha_0}}-\alpha_0 x+\frac{\alpha_0}{x}.$

Ta có

$$f'(x)=\alpha_0 (x^{\alpha_0-1}-1)\left(1-\frac{1}{x^{\alpha_0+1}}\right)\ge 0 \forall x\ge 1,$$

nên $f(x)\ge f(1)=0.$

Bước 2: Chứng minh $\alpha=\sqrt{3/2}=\alpha_0$ là hằng số nhỏ nhất. Thật vậy, giả sử tồn tại $1<\alpha<\alpha_0$ sao cho bất đẳng thức ở đề bài thỏa mãn với mọi $x\ge 1.$ Khi đó tồn tại $m>n$ nguyên dương sao cho $\alpha<\frac{m}{n}<\alpha_0.$ Khi đó $h(\alpha)<h\left(\frac{m}{n}\right).$ Từ đây suy ra

$$2h\left(\frac{m}{n}\right)+2\ge3h(1),$$

tương đương

$$\frac{2(y^m-1)^2}{y^m}\ge\frac{3(y^n-1)^2}{y^n} \, \forall y\ge 1,$$

với $y=\sqrt[n]{x}.$ Bất đẳng thức trên có thể được viết lại với $y>1$ như sau

$$g(y)=2(y^{m-1}+y^{m-2}+\cdots+1)^2-3y^{m-n}(y^{n-1}+y^{n-2}+\cdots+1)^2\ge 0, \, \forall y>1.$$

Vì $g(y)$ liên tục nên ta có

$$\lim_{y\to 1}g(y)\ge0 \iff 2m^2\ge3n^2,$$

mâu thuẫn do $m/n<\sqrt{3/2}.$

Vậy hằng số nhỏ nhất cần tìm là $\sqrt{3/2}.$

Bài 6.

Đặt $a = 3^{4(n^2+n+1)} \ge 81$ thì ta có $A=t^7+t^5+1 $. Phân tích biểu thức này thành nhân tử, ta có

$$A=(t^5-t^4+t^3-t+1)(t^2+t+1).$$

Dễ dàng chứng minh được $$t^5-t^4+t^3-t+1 > t^2+t+1 > 1$$ nên biểu thức trên không thể là số nguyên tố được.

Vậy không tồn tại số $n$ nào thỏa mãn đề bài.

Bài 7.

Trong $(i),$ thay $m=1$, ta có

$$f(f(n))=n^6f(n).(*)$$

Từ đây dễ suy ra $f$ đơn ánh. Tiếp tục thay $m$ bởi $f(m),$ ta có

$$f(f(m) \cdot f(n))=n^6 \cdot f(f(m) \cdot n)=(mn)^6 \cdot f(mn)=f(f(mn)).$$

Kết hợp với tính đơn ánh, ta có $ f(m) \cdot f(n)=f(mn)$ nên $f$ nhân tính. \medskip

Tiếp theo, giả sử tồn tại $n$ sao cho $\gcd (f(n),n)=1$ suy ra $f(f(n))$ và $f(n)$ nguyên tố cùng nhau. Từ $(*)$ ta được $f(n)=1$ và $f(1)=n^6$. Lại có $\gcd (f(1),1)=1$ nên $f(1)=1$ . Do đó $$\gcd (f(n),n) \neq 1,\forall n \ge 2.$$

Xét số nguyên $n$ bất kì. Giả sử $f(n)$ có ước nguyên tố $q$ sao cho $q$ không là ước của $n.$ Khi đó $\gcd (n,q)=1$ và $\gcd (f(n),f(q)) \neq 1, $ trái với điều kiện $(ii).$ \medskip

Xét số nguyên tố $p$ bất kì. Khi đó $f(p)=p^k, k \ge 1$ do $(f(p),p) \neq 1$. Suy ra $$f(f(p))=f(p^k)=f(p)^k= p^{k^2}.$$

Thay vào $(*)$ ta được $$p^{k^2}=p^6 \cdot p^k \Rightarrow \begin{cases} k=3 \\ f(p)=p^3 \end{cases}.$$

Từ đây, kết hợp với tính nhân tính của $f$ và ta cũng biết rằng mỗi số $n$ đều có thể biểu diễn thành tích của các số nguyên tố, ta có được $f(n)=n^3$ với mọi $n \in \mathbb{N}.$

Bài 8.

Giả sử $MF$ cắt $BC$ tại $T$, ta có $(C)$ tiếp xúc với $BC$ tại $H$ nên $$TH^2=TF \cdot TM.$$

Không có mô tả.

 

Ta lại có $NQ \parallel BC$ vậy nên $$\angle{TBF}=\angle{FQN}=\angle{TMB},$$ suy ra $TB^2=TF \cdot TM.$ Từ đó, suy ra $T$ là trung điểm $HB$. Tương tự, ta cũng chứng minh được rằng $NE$ cắt $BC$ cũng tại trung điểm $BH$.

Suy ra $NE, MF, BC$ đồng quy tại $T$ là trung điểm $BH$, là một điểm cố định.

Nhận xét: Bài toán này cũng có một phiên bản tương tự khi thay tam giác tam giác cân bởi tam giác bất kỳ, và đường tròn cắt tại hai điểm trên cạnh thành đường tròn nội tiếp. Cụ thể là:

Cho tam giác $ABC$ có đường tròn nội tiếp $(I)$ tiếp xúc với các cạnh $BC,CA,AB$ lần lượt tại $D,E,F.$ Đường thẳng qua $E,F$ song song với $BC$ lần lượt cắt lại $(I)$ tại $G,H$. Giả sử $BG,CH$ cắt lại $(I)$ tại $P,Q$ và $EF$ cắt $BC$ tại $T$.

a/ Chứng minh rằng $PE,QF$ cùng đi qua trung điểm $K$ của $DT.$

b/ Chứng minh rằng $(KBP),(KCQ)$ cùng tiếp xúc với $(I)$ và hai đường tròn này cũng cắt nhau tại một điểm nằm trên $(ABC)$.

Đề thi chọn đội tuyển PTNK năm học 2010-2011

Ngày thi thứ 1

Bài 1.  Giải hệ phương trinh: $\left\{\begin{array}{l}\frac{5(x+y)}{x+y+6 x y}+\frac{6(x+z)}{x+z+5 x z}=4 \\ \frac{6(z+y)}{z+y+4 z y}+\frac{4(x+y)}{x+y+6 x y}=5 \\ \frac{4(x+z)}{x+z+5 x y}+\frac{5(y+z)}{y+z+4 y z}=6\end{array}\right.$

Bài 2.  Tìm tất cả các hàm $f: \mathbb{R} \rightarrow \mathbb{R}$ thỏa mãn

$f(|x|+y+f(y+f(y)))=3 y+|f(x)|$ với mọi $x, y \in \mathbb{R}$

Bài 3.  Cho $p$ là số nguyên tố lẻ và $n=2 p+r$ với $r \in{0,1,2, \ldots, p-1} .$ Đặt $X={1,2, \ldots, n}$. Ánh xạ $f: X \rightarrow X$ được gọi là có tính chất $P$ nếu $f$ không phải là ánh xạ đồng nhất và $f(f \ldots(f(f(k))) \ldots)=k$ (ánh xạ hợp plần) với mọi $k \in X .$ Đạt $A_{f}={k \in X \mid f(k)=k}$.

  1. Chứng minh rằng nếu $f$ có tính chất P thì $\left|A_{f}\right| \equiv r(\bmod p)$.
  2. Gọi $d$ là số các ánh xạ $ f$ có tính chất $P$. Chứng minh rằng $d$ không là ước số của n!.

Bài 4.  Cho tam giác $A B C$ nội tiếp đường tròn $(O)$ có $A$ cố định và $B, C$ thay đổi trên $(O)$ sao cho $B C$ luôn song song với một đường thẳng cố định. Các tiếp tuyến của $(O)$ tại $B$ và $C$ cắt nhau tại $K$. Gọi $M$ là trung điểm $B C, N$ là giao điểm của $A M$ với $(O)$. Chứng minh rằng đường thẳng $K N$ luôn đi qua một điểm cố định.

Ngày thi thứ 2

Bài 5.  Cho $a, b, c$ là độ dài các canh của một tam giác. Chứng minh rằng $(2 a+2 b-c)(2 b+2 c-a)(2 a+2 c-b)>25 a b c$

Bài 6.  Cho dãy số $u_{n}$ thỏa mãn $u_{1}=\sqrt{2}, u_{n+1}=\frac{2 u_{n}^{2}+5 u_{n}+5}{2 u_{n}+4}, n \geq 1$. Tính giới hạn sau $\lim \frac{u_{n}^{2}-3 u_{n}+5}{3 n^{2}+4 n-1}$.

Bài 7.  Xét số tự nhiên $n>1$. Bắt đầu từ bộ số $1,2, \ldots, 2 n-1,2 n$ ta thực hiện phép biến đổi sau: chọn hai số $a, b$ bát kì sao $a-b>1$, xóa hai số này và thay bởi hai số $a-1$ và $b+1$. Với bộ số mới này ta lại tiếp tục thực hiện phép biến đổi tương tự.

  1. Chứng minh rằng ta sẽ đạt đến trạng thái dừng, tức là không thể thực hiện phép biến đổi như vậy được nữa.
  2. Gọi $k$ là số lần phép biến đổicần thực hiện để đạt đến trang thái dừng. Tìm giá trị nhỏ nhất và lớn nhất của $k$.

Bài 8.  Cho đường tròn $\left(\gamma_{1}\right)$ dường kính $A B$ và đường tròn $\left(\gamma_{2}\right)$ tâm $A$ cắt $\left(\gamma_{1}\right)$ tai $C$ và $D$. Điểm $M$ thay đổi trên cung $C D$ (nằm bên trong $\left.\left(\gamma_{1}\right)\right)$ của $\left(\gamma_{2}\right)$, $B M$ cắt $\left(\gamma_{2}\right)$ tại $N$ (N khác $M$ và B). Tìm giá trị nhỏ nhất của $\frac{N C+N D}{N M}$.

Lời giải

Bài 1. Đặt $u=\frac{x+y}{x+y+6xy},v=\frac{y+z}{y+z+4yz},w=\frac{z+x}{z+x+5zx}$ thì ta có hệ
$$\left\\{ \begin{aligned}
& 5u+6w=4 \\\\
& 6v+4u=5 \\\\
& 4w+5v=6 \\\\
\end{aligned} \right.\Leftrightarrow \left\\{ \begin{aligned}
& 8u=1 \\\\
& 4v=3 \\\\
& 16w=9 \\\\
\end{aligned} \right..$$
Suy ra
$$\left\\{ \begin{aligned}
& 7(x+y)=6xy \\\\
& 3(y+z)=12yz \\\\
& 7(z+x)=45zx \\\\
\end{aligned} \right.\Leftrightarrow \left\\{ \begin{aligned}
& a+b=\frac{6}{7} \\\\
& b+c=12 \\\\
& c+a=\frac{45}{7} \\\\
\end{aligned} \right.,$$ trong đó $a=\frac{1}{x},b=\frac{1}{y},c=\frac{1}{z}.$
Giải hệ trên, ta thu được $a=-\frac{33}{14},b=\frac{45}{14},c=\frac{123}{14}$ nên $(x,y,z)=\left( -\frac{14}{33},\frac{14}{45},\frac{14}{123} \right).$

Bài 2.

Dễ thấy $f$ toàn ánh. Giả sử $f(a)=0$ và thay $x=0,y=a,$ ta có
$$0=3a+\left| f(0) \right|.$$
Suy ra $a$ tồn tại duy nhất và $a=-\frac{1}{3}\left| f(0) \right|\le 0.$ Lại thay $x=y=a,$ ta có $f(0)=3a\le 0.$
Lại thay $x=-a,y=a$ thì chú ý rằng $\left| -a \right|+a=0$, ta có $f(0)=3a+\left| f(-a) \right|$ nên $f(-a)=0$, điều này kéo theo $a=-a$ hay $a=0$ (do tính duy nhất ở trên). \medskip

Thay $y=0$ thì $f(\left| x \right|)=\left| f(x) \right|$ nên $f(x)\ge 0,\forall x\ge 0.$
Xét $x>0$ và $y=-\frac{f(x)}{3}$, ta có $f\left( x-\frac{f(x)}{3}+f\left( -\frac{f(x)}{3}+f\left( -\frac{f(x)}{3} \right) \right) \right)=0$ nên
$$-\frac{f(x)}{3}+f\left( -\frac{f(x)}{3}+f\left( -\frac{f(x)}{3} \right) \right)=-x$$ với mọi $x>0.$
Trong đề bài, thay $x=0$ thì $f(y+f(y+f(y)))=3y$. Thay $y\to -\frac{f(x)}{3}$ thì
$f\left( -\frac{f(x)}{3}+f\left( -\frac{f(x)}{3}+f\left( -\frac{f(x)}{3} \right) \right) \right)=-f(x)$.
So sánh hai đẳng thức trên, ta có $f(-x)=-f(x),\forall x>0$ nên $f$ là hàm số lẻ. \medskip

Từ tính chất hàm số lẻ, ta có $f\left( \frac{f(x)}{3}+f\left( \frac{f(x)}{3}+f\left( \frac{f(x)}{3} \right) \right) \right)=f(x)$ với mọi $x>0.$
Trong đề bài, xét $x\ge 0$ và $y\to \frac{f(y)}{3}$, ta có
$$f\left( x+\frac{f(y)}{3}+f\left( \frac{f(y)}{3}+f\left( \frac{f(y)}{3} \right) \right) \right)=f(y)+f(x)$$ hay
$f(x+y)=f(x)+f(y)$ với mọi $x,y>0.$
Vì $f$ cộng tính trên ${{\mathbb{R}}^{+}}$ nên ta có $f(x)=ax,\forall x>0.$ Lại do tính chất hàm lẻ, ta suy ra $f(x)=ax,\forall x\in \mathbb{R}.$ Thay vào đề bài, ta có $a=1.$ \medskip

Vậy tất cả các hàm số cần tìm là $f(x)=x.$

Bài 3.

a) Ta có
$$\left | A_{f} \right |\equiv r \pmod{p} \Leftrightarrow \left | X\setminus A_{f} \right | \text{ chia hết cho } p.$$
Điều này tương đương số phần tử của tập hợp $B= \left \\{ k\in X|f(k)\neq k \right \\}$ là bội của $p.$
Đặt $f_{m}(x)$ là ánh xạ hợp $m$ lần. Xét $x\in B$ thì cũng có các số $f(x),f_{2}(x),\ldots, f_{p-1}(x)\in B$. Thật vậy, \medskip

Gỉa sử tồn tại $1<m<p$ sao cho $f_{m}(x) =x$ với số $x \in B$ nào đó, ta chọn $m$ là số nhỏ nhất như thế. Vì $p$ nguyên tố lẻ nên $p$ không chia hết cho $m.$ Do vậy tồn tại số $t$ sao cho $0<p-tm<m$. Lại có $$f_{m}(x) =x\Rightarrow f_{tm}(x)=x\Rightarrow f_{p-tm}(x)=f_{p}(x)=x$$ (mâu thuẫn với tính nhỏ nhất của $m$). Vì thế nên với mọi $m$ mà $1<m<p $ thì $f_{m}(x)\neq x$. Từ đó suy ra với mọi $1<k<l<p$ thì $f_{k}(x)\neq f_{l}(x) $, tức là $x,f(x),f_{2}(x),\ldots, f_{p-1}(x) $ là $p$ số khác nhau thuộc $B.$ \medskip

Xét số $y \in B$ và $y$ khác tất cả $p$ số ở trên. Khi đó, ta cũng sẽ có $y$ sinh ra một bộ $p$ số phân biệt mới. Giả sử rằng có $f_{i}(x)=f_{j}(y)$ với $i<j$ nào đó thì sẽ có $f_{p+i-j}(x)=f_{p}(y)=y $, mâu thuẫn. Suy ra trong $B$ sẽ có $1$ hoặc $2$ bộ $p$ số rời nhau, chứng tỏ rằng số phần tử của $B$ chia hết cho $p.$ Suy ra đpcm. \medskip

(b) Từ đây ta thấy rằng để đếm số ánh xạ $f$ có tính chất $\mathcal{P},$ trước hết, ta chọn ra $r$ hoặc $p+r$ vị trí cố định. Ta xét hai trường hợp như sau:

Nếu $\left| A_f \right|=p+r$ thì có $C_n^{p+r}$ cách chọn ra các số này, còn lại $p$ số thì $f$ phải là song ánh trên tập con đó. Do đó trong trường hợp này có $p!C_n^{p+r}$ cách.
Nếu $\left| A_f \right|=r$ thì tương tự trên, ta cũng đếm được $(p!)^2C_n^rC_{2p}^p$.

Từ đó suy ra số ánh xạ tính chất $\mathcal{P}$ là $$d=p!C_{n}^{p+r}+{{(p!)}^{2}}C_{n}^{r}C_{2p}^{p}.$$ Ta sẽ chứng minh số này không là ước của $n!.$ Ta viết số $d$ dưới dạng khai triển
$$d=p!\frac{n!}{(p+r)!p!}+{{(p!)}^{2}}\frac{n!}{r!(2p)!}\cdot \frac{(2p)!}{{{(p!)}^{2}}}=\frac{n!}{(p+r)!}+\frac{n!}{r!}.$$
Đặt $(p+r)!=k\cdot {{(r!)}^{2}}$ với $k=\frac{(p+r)!}{{{(r!)}^{2}}}=\frac{p!}{r!}\cdot \frac{(p+r)!}{p!r!}=\frac{p!}{r!}C_{p+r}^{r}\in \mathbb{Z}$. Khi đó, ta viết lại
$$\frac{n!}{d}=\frac{r!(p+r)!}{r!+(p+r)!}=\frac{k\cdot {{(r!)}^{3}}}{(1+k\cdot r!)\cdot r!}=\frac{k\cdot {{(r!)}^{2}}}{k\cdot r!+1}.$$
Dễ thấy số này không thể nguyên vì $k\cdot r!+1$ nguyên tố cùng nhau với $k\cdot {{(r!)}^{2}}$. Từ đó ta có $d$ không là ước của $n!.$

Bài 4. Giả sử $KN$ cắt $(O)$ tại $I$ thì tứ giác $BNCI$ điều hòa.

Do đó $A(BC,NI)=-1,$ mà $AN$ chia đôi $BC$ nên $AI\parallel BC,$ tức là $AI$ có phương cố định. Từ đó ta thấy $I$ là điểm cố định cần tìm.

Bài 5. Đặt $a+b-c=x,b+c-a=y,c+a-b=z$ thì $x,y,z>0.$ Ta đưa về bất đẳng thức
$$\left( 4\cdot \frac{x}{y+z}+1 \right)\left( 4\cdot \frac{y}{z+x}+1 \right)\left( 4\cdot \frac{z}{x+y}+1 \right)>25.$$
Không mất tính tổng quát, giả sử $0<x\le y\le z.$ Đặt $S=x+y+z.$ Ta đưa về
$$(S+3x)(S+3y)(S+3z)>25(S-x)(S-y)(S-z).$$
Khai triển và rút gọn, ta được
$${{S}^{3}}-4S(xy+yz+zx)+13xyz>0.$$
Chú ý rằng $${{S}^{3}}-4S(xy+yz+zx)=S({{S}^{2}}-4(xy+yz+zx))=S({{(x+y-z)}^{2}}-4xy)$$ nên ta đưa về $S{{(x+y-z)}^{2}}+xy(13z-4S)>0$.
Bất đẳng thức cuối đúng vì $13c-4S=9z-4(x+y)>0.$

Bài 6.

Lời giải. Ta thấy rằng $u_{n}>0, \forall n$ và $u_{n+1}-u_{n}=\frac{u_{n}+5}{2\left(u_{n}+2\right)}>0$ nên dãy tăng. Giả sử dãy bị chặn trên thì nó hội tụ về $L>0$, suy ra
$$
L=\frac{2 L^{2}+5 L+5}{2 L+4} \Leftrightarrow L=-5
$$
vô lý. Suy ra $\lim_{n \rightarrow+\infty} u_{n}=+\infty$. Từ đó, ta được
$$
\lim_{n \rightarrow+\infty}\left(u_{n+1}-u_n\right)=\lim_{n \rightarrow+\infty} \frac{u_n+5}{2\left(u_n+2\right)}=\frac{1}{2}
$$
nên theo định lý Stolz, ta suy ra $\lim_{n \rightarrow+\infty} \frac{u_n}{n}=\frac{1}{2}$ và $\lim_{n \rightarrow+\infty} \frac{u_n}{n^2}=0 .$ Do đó, trong biểu thức cần tính giới hạn, chia tử và mẫu cho $n^2$ rồi áp dụng kết quả trên, ta có
$$
\lim_{n \rightarrow+\infty} \frac{u_n^2-3 u_n+5}{3 n^2+4n-1}=\lim _{n \rightarrow+\infty} \frac{\left(\frac{u_n}{n}\right)^{2}-\frac{3 u_n-5}{n^2}}{3+\frac{4}{n}-\frac{1}{n^2}}=\left(\frac{1}{2}\right)^{2} \cdot \frac{1}{3}=\frac{1}{12}
$$

Bài 7.

(a) Xét đại lượng $S$ là tổng bình phương các số thu được sau mỗi thao tác biến đổi. \medskip

Ta thấy rằng từ $(a,b)$ với $a-b>1,$ ta đưa về bộ $(a-1,b+1)$ thì tổng trên thay đổi một lượng là
${{a}^{2}}+{{b}^{2}}-{{(a-1)}^{2}}-{{(b+1)}^{2}}=2(a+b-1)>0.$
Do đó, tổng $S$ giảm ngặt, và rõ ràng $S$ phải luôn dương nên thao tác trên chỉ thực hiện được trong hữu hạn lần.
\medskip

(b) Rõ ràng tổng trên không đổi khi không còn cặp số $a,b$ nào mà $a-b>1.$ Điều này đồng nghĩa với việc các số thu được trong trạng thái cuối chỉ nhận hai giá trị liên tiếp nào đó. Ta thấy rằng tổng các số đã cho luôn không đổi và là $1+2+\cdots +2n=n(2n+1).$ \medskip

Giả sử cuối cùng, ta có $x$ số $m$ và $y$ số $m+1$ thì
$$\left\\{ \begin{aligned}
& x+y=2n \\\\
& mx+(m+1)y=n(2n+1) \\\\
\end{aligned} \right..$$
Suy ra $2mn+y=2{{n}^{2}}+n\Rightarrow n|y.$ Tuy nhiên, nếu $y\in \{0,2n\}$ thì vô lý vì vế phải không chia hết cho $2n.$ Do đó $x=y=n$ và $m=n,$ tức là ở trạng thái cuối, ta còn $n$ số $n$ và $n+1.$

  • Tổng bình phương của chúng là $S=n\cdot {{n}^{2}}+n\cdot {{(n+1)}^{2}}=n(2{{n}^{2}}+2n+1).$
  • Tổng bình phương ban đầu là ${{S}_{0}}={{1}^{2}}+{{2}^{2}}+\cdots +{{(2n)}^{2}}=\frac{n(2n+1)(4n+1)}{3}.$

Suy ra ${{S}_{0}}-S=\frac{2}{3}({{n}^{3}}-n).$ \medskip

(b) Để thực hiện được nhiều lần nhất thì giá trị giảm đi ở mỗi lần phải ít nhất. Theo câu a) thì giá trị đó sẽ là $2(a+b-1)\ge 2.$ \medskip

Suy ra số lần nhiều nhất sẽ là $\frac{1}{3}({{n}^{3}}-n)$. Để thực hiện được điều này, ta sẽ cố gắng trong mỗi thao tác tạo ra nhiều giá trị nhất có thể và đồng thời làm giảm số lượng các giá trị ở hai biên đi. Từ đó ta được ${{k}_{\max }}=\frac{1}{3}({{n}^{3}}-n).$ \medskip

Để thực hiện được ít lần nhất, ta sử dụng ý tưởng tham lam, mỗi lần, ta sẽ chọn các cặp số nằm về hai phía của $n,n+1.$ Khi đó, giá trị của các số $1,2,\ldots ,n-1$ sẽ dần dần được tăng lên, trong khi giá trị của các số $n+2,n+3,\ldots ,2n$ dần dần sẽ giảm đi. Tổng khoảng cách từ các số nhỏ hơn $n$ đến $n$ là $1+2+\cdots +n-1=\frac{n(n-1)}{2}$. Tương tự thì tổng khoảng cách các số lớn hơn $n+1$ đến $n+1$ cũng là $\frac{n(n-1)}{2}$. Ta thấy mỗi lần thao tác thì các số này sẽ thu hẹp khoảng cách đúng $2$ đơn vị nên số lần thao tác tối thiểu phải là $\frac{1}{2}\left( \frac{n(n-1)}{2}+\frac{n(n-1)}{2} \right)=\frac{n(n-1)}{2}.$ \medskip

Để đạt được giá trị này, mỗi lần, ta chỉ cần chọn các cặp số có dạng $(t,2n+1-t)$ với $1\le t\le n-1$ là được. Suy ra ${{k}_{\min }}=\frac{n(n-1)}{2}.$

Bài 8.

Theo định lý Ptolemy cho tứ giác $BCND$ nội tiếp trong ${{\gamma }_{1}}$ thì
$$BC\cdot ND+BD\cdot NC=BN\cdot CD.$$
Vì $AC=AD$ nên $BC=BD=m$ và $CD=n$ là các giá trị cố định.

Ta có
$$m(NC+ND)=n\cdot BN\Rightarrow NC+ND=\frac{n}{m}\cdot BN.$$
Suy ra $\frac{NC+ND}{MN}=\frac{n}{m}\cdot \frac{BN}{MN}.$ Ta đưa về tìm giá trị nhỏ nhất của $\frac{BN}{MN}.$ \medskip

Xét phương tích từ $B$ đến ${{\gamma }_{2}}$ thì $BM\cdot BN=BK\cdot BA=c$ là hằng số nên
$(BN-MN)BN=c$.
Do đó $\frac{MN}{BN}=1-\frac{c}{B{{N}^{2}}}$ nên
$$\frac{BN}{MN}\min \Leftrightarrow \frac{MN}{BN}\max \Leftrightarrow \frac{c}{B{{N}^{2}}}\min \Leftrightarrow B{{N}^{2}}\max .$$
Dễ thấy $\max BN=AB$, xảy ra khi $N\equiv A$ hay $M\equiv K.$ Khi đó $\frac{NC+ND}{MN}=\frac{AC+AD}{AK}=2$ chính là giá trị nhỏ nhất cần tìm.